Bar Prep - MCQ Diagnostic

अब Quizwiz के साथ अपने होमवर्क और परीक्षाओं को एस करें!

On February 1, the owner of a parcel delivered a deed to a woodcutter granting him a profit in gross in the land. As holder of the profit, the woodcutter was entitled to enter the land for the purposes of cutting and taking any timber growing on the property. On April 1, the owner entered into a contract to sell the land to a buyer. On April 15, in exchange for $50,000 the owner delivered a warrant deed for the land to the buyer. On April 30, the woodcutter properly recorded his interest in the land. The buyer did not discover the woodcutter's interest in the land until May 2, when the buyer happened to notice the woodcutter leaving the land driving a truck loaded with cut timber. That same day, the buyer recorded his deed to the land. Under which type of recording act will the buyer prevail? A. A notice recording act. B. A race recording act. C. A race-notice recording act. D. All of the above, because the woodcutter's profits in gross cannot be protected by a recording act.

A. A notice recording act. Discussion of correct answer:Under the common law, priority is generally awarded to the grantee who was first to receive an interest from the grantor (this is known as "first-in-time, first-in-right"). A recording act may alter the outcome, depending on whether it is a race, notice, or race-notice type of act. Under a notice-type recording act, a subsequent bona fide purchaser who takes for value and without notice of the prior conveyance will prevail over a grantee who failed to record his interest prior to the date of the conveyance to the subsequent purchaser. The buyer purchased the land without notice of the woodcutter's interest, because the woodcutter did not record the profit until after the conveyance. Thus, the buyer will prevail under a notice recording act, despite the fact that he recorded after the woodcutter did so.

Walking by a stranger's home one morning, a young man spied a pearl necklace through the window. Thinking that the necklace would have a good pawnshop value and that he could use some quick money, the young many came up with a plan to steal it. He knocked not eh front door of the house. When the homeowner answered, the young man falsely told her that his car broke down and asked to use her phone. The sympathetic homeowner agreed and went to get the phone. While the homeowner was out of the room, the young man took the necklace. When the homeowner returned with the phone, the young man made a call, thanked the homeowner, and left. The young man then went directly to the nearest pawnshop, where he sold teh necklace. Of what crime(s), if any, is the young many guilty of? A. Larceny and burglary. B. Larceny, but not burglary. C. Burglary, but not larceny. D. Neither, larceny nor burglary.

A. Larceny and burglary. Discussion of correct answer:Larceny is defined as the trespassory taking ("caption") and carrying away ("asportation") of the property of another with the intent to permanently deprive the owner thereof. Here, the young man wrongfully took the pearl necklace belonging to the homeowner, and, thus, a trespassory taking occurred. The young man then left with the pearl necklace, satisfying the "carrying away" element of the crime. Finally, the young man sold the necklace, an act indicating his intention to permanently deprive the homeowner of the necklace. As such, the elements of the crime of larceny are satisfied, and the young man is guilty of this crime. Burglary at modern law is the breaking into and entering of the structure of another with the intent to commit a felony or theft offense therein. Here, the young man used a false premise (a broken-down car) to gain entry to the homeowner's dwelling so that he could steal the pearl necklace. Gaining entry to the homeowner's home by means of a fraudulent claim amounts to a constructive breaking, satisfying the breaking element of burglary, and the young man did indeed intend to commit a larceny therein. As such, the requisite elements of burglary are present, making the young man guilty of larceny as well as burglary.

Which evidence is admissible as proper lay opinion? A. Testimony of a witness who will testify that the red car sped through the light. B. The testimony of a recent high school graduate who will testify that the plaintiff was an alcoholic. C. The testimony of ac computer engineer, who will testify that the spam email came from the plaintiff's computer. D. Any evidence likely to assist the trier of fact.

A. Testimony of a witness who will testify that the red car sped through the light. Discussion of correct answer:The proper scope of non-expert opinion includes such perceptions as speed and other measurements and physical states such as intoxication or injury. Legal conclusions, however, must be avoided. For example, the lay witness cannot testify that an individual was schizophrenic, alcoholic, or that an accident victim fractured his spine.

Two women, twin sisters, hav just graduated from college. As a graduation gift, their parents purchased a house for them to share in a quiet neighborhood in the suburbs of their hometown. The twins were especially excited to see that the house has a large fenced backyard where they can play with their chocolate Labrador retriever puppies. One sunny afternoon, while the twins were playing Frisbee with one of their dogs, one of the women overthrew the Frisbee, causing it to land in the neighbor's yard. The neighbor had just finished grilling gourmet cuts of steak for his family and had placed them on a heavy platter. As the neighbor was carrying the platter towards his house, the Frisbee flew by his head. Startled, he dropped the platter of steaks. The platter shattered and cut deeply into his foot, requiring him to make a trip to the hospital for stitches. The neighbor sued the woman for assault and battery. What is the likely result? A. The neighbor will lose on both claims. B. The neighbor will prevail on both claims. C. The neighbor will prevail on the assault claim, but not the battery claim. D. the neighbor will prevail on the battery claim, but not the assault claim.

A. The neighbor will lose on both claims. Discussion of correct answer:Battery is defined as an act by a defendant that causes harmful or offensive contact with the person of another where the defendant intended to cause the harmful or offensive contact. Assault is defined as an act by the defendant, intended to cause the plaintiff to apprehend an imminent harmful or offensive contact to the plaintiff's person that causes reasonable apprehension of the contact. Both torts require intent on the part of the defendant. Neither battery nor assault would be the proper cause of action, because the woman did not intend to cause either a harmful or offensive touching or an imminent apprehension of a harmful or offensive touching to the neighbor's person.

A man was driving when his brakes failed. His car hit a teenager riding his bike and a woman out for a walk before the car came to a full stop. The teenager sued the man for negligence. He recovered a default judgment for the full amount of his claim. Which of the following statements is correct? A. The teenager's judgment should have no effect upon a subsequent lawsuit by the woman against the man. B. The teenager's judgment is res judicata with respect to a subsequent lawsuit by the woman against the man. C. Assuming the mutuality doctrine is inapplicable, the court may permit the woman to assert collateral estoppel in a subsequent lawsuit by the woman against the man. D. Because the woman and the teenager are in privity, the woman cannot recover because of the res judicata effects of the initial judgment for the teenager.

A. The teenager's judgment should have no effect upon a subsequent lawsuit by the woman against the man. Discussion of correct answer:Because the teenager recovered a default judgment against the man in the initial lawsuit, it would have no collateral estoppel effect (i.e., obviously, the issues were not actually litigated). Also, because the woman and the teenager are not in privity, the initial judgment would have no res judicata effect.

After a car accident, a woman from State A consults an attorney. The attorney indicated to the woman that they had a good claim against State B defendant. However, the attorney wanted the woman to see a physician to help in assessing damages. The physician told the layer that the woman's injuries were easily in excess of $100,000. The lawyer then filed a negligence claim against the defendant in federal court. Both State A and B recognize the physician patient privilege. At trial, the defendant wanted to call the physician, who did not testify for the plaintiff, to testify as to a conversation the physician and the plaintiff had while the plaintiff was in the physician's office. Only the physician, plaintiff, and the physician's assistant were present. Should the physician's testimony be admitted? A. Yes, because the physician patient privilege does not apply. B. Yes, because the physician patient privilege was broken due to the fact that the physician's assistant was in the room. C. No, because the physician's assistant's presence did not break the physician patient privilege. D. No, because the patient, not the physician, is the holder of the privilege.

A. Yes, because the physician-patient privilege does not apply. Discussion of correct answer:Generally, if a state has a physician-patient privilege, and the case is being heard in federal court based on diversity, then the federal court will apply the privilege. However, there are situations where the privilege does not exist, such as when the patient is being examined by the physician for something other than treatment. Here, the physician was being used by the lawyer to assess the amount in controversy in order to satisfy the requirement for diversity jurisdiction. Therefore, the privilege does not arise, and the physician may testify.

Which of the following actions by the federal government is most likely to be found unconstitutional? A. Congressional enactment of legislation prohibiting the sale of military systems software technology to the People's Republic of China. B. A joint resolution fo the houses of Congress resolving a dispute between State Red and State Blue regarding the ownership of the island situation between the two states. C. Assertion of appellate jurisdiction by the US Supreme Court over a case decided by a state's Supreme Judicial Court. D. A decision by the Senate to reject an African American nominee to the US Circuit Court of Appeals for the Ninth Circuit.

B. A joint resolution of the houses of Congress resolving a dispute between State Red and State Blue regarding the ownership of the island situated between the two states. Discussion of correct answer:Congress has no authority to resolve disputes between states; that authority rests exclusively in the judicial branch of government. Under the power granted in Article III of the Constitution, the U.S. Supreme Court has original jurisdiction in all cases "in which a State shall be a party," and has appellate jurisdiction in all cases arising "between two or more states." The dispute over ownership of the island is a dispute between two states that must be resolved in the Supreme Court, not in Congress

Every summer, a nonprofit art gallery held a major fundraising event. Although there were many aspects to the fundraiser, the key earner for the gallery was the silent auction. Local businesses donated services and merchandise for "purchase" by local residents. Each year, a seafood restaurant, a well-known local resident, donated a dinner for 10 people. Often, local organizations and businesses would bid on the dinner and used the package to host an awards dinner or entertain clients. This year, a local business submitted the winning bid of $1,500 for the dinner package. Unfortunately , after the auction but before the payment had been exchanged, the building in which the seafood restaurant operated was determined to be structurally unsound, and the building inspector shut down the restaurant indefinitely. If the local business refuses to make payment for dinner for 10 ,does the gallery have a valid claim against the business? A. No, because the local business' promise to pay was an executory gift. B. No, because the gallery is not able to supply a dinner for 10 people at the seafood restaurant. C. Yes, because the seafood restaurant's difficulties excuse the gallery's performance, but not the local business' performance. D. Yes, because the local business' promise was made for charitable purposes.

B. No, because the gallery is not able to supply a dinner for 10 people at the seafood restaurant. Discussion of correct answer:If contractual duties become impossible to perform after a contract has been entered into by the parties, the parties' contractual duties will be discharged. Contractual duties may become impossible to perform if the subject matter of the contract or the means of performing are destroyed. For a duty to perform to be discharged, the thing destroyed must be essential to performance. Here, it became impossible for the gallery to perform, and the gallery's duty to perform was discharged. By the same token, the local business' performance was also excused, and the gallery cannot recover from the business.

A man was convinced by his friends to attend a dogfight, even though dogfighting is illegal throughout the US. The man watched and gambled on the first four fights, which left many of the dogs maimed and bloody. Hoping to get a closer look at the next fight, the man moved up to the front row. During the next fights dog broke free form the ring and ran through the crowd. In the frenzy, the dog bit the man, leaving him with multiple wounds and permanent scars. The man filed a strict liability suit against the dog's owner for his injuries. Will the injured man prevail? A. No, because it was not foreseeable that the dog would jump out of the ring. B. No, because the man assumed the risk of injury. C. Yes, because dogfighting is illegal. D. Yes, because the owner failed to properly protect the public.

B. No, because the man assumed the risk of injury. Discussion of correct answer:The owner of an animal with known dangerous propensities is generally strictly liable for harm caused by the animal. However, a plaintiff is completely barred from recovery in strict liability if it can be shown that he assumed the risk of injury. To assume the risk of injury, a plaintiff must know of and appreciate the danger and voluntarily expose himself to such danger. Here, the man knew that dogfights involve dogs that are taught to inflict serious injury by biting. Despite this knowledge, the man not only attended the event, but also moved closer to the ring to get a better view. As such, the man assumed the risk of harm and, therefore, cannot recover under strict liability for his injuries.

On Feb. 1, a woman contracted with a famous landscaper to drastically change the sprawling front lawn of her mansion. The woman had long admired the landscapers work. The landscaper promised to complete the work before April 30, the date the woman would host her annual spring party. On March 1, the woman left for a month vacation. After beginning the work on the woman's lawn, the landscaper became increasingly anxious about finishing other prior commitments and assigned the contract to a local lawn care professional. When the woman returned, she was horrified to find the lawn care professional shaping the hedges of her front lawn. The woman contacted the landscaper, and he informed her of the assignment to the lawn care professional and told her that he was no longer obligated under the original contract. Although the landscaping project was coming along nicely, the woman fired the landscaper anyway. If the woman asserts a claim against the landscaper based on breach of contract, what effect with the fact that the landscaper assigned the contract to the lawn care professional have? A. It will relieve the landscaper of liability, because the contract was assignable. B. It will relieve the landscaper of liability, because the landscaping project was satisfactory to the woman thus far, and she suffered no damages. C. It will not relieve the landscaper of liability, because a personal services contract of this nature is non-delegable. D. It will not relieve the landscaper of liability, because the landscaper did not inform the woman of the assignment in advance.

C. It will not relieve the landscaper of liability, because a personal services contract of this nature is non-delegable. Discussion of correct answer:Although contracts are generally assignable and delegable, those involving personal services are not. In this case, the parties' contract is non-delegable, because the woman hired the landscaper for the landscaper's specific talents. Therefore, this answer choice is correct.

A father was accused of manslaughter after beanie a man to death on the side of the road. The confrontation occurred after the man, who was drunk, hit the father's car on the highway with the father's newborn baby in the backseat. The prosecutor submitted the information to the grand jury, but the grand jury failed to indict the father. When the prosecutor later learned that the father and the man were in a legal battle over a piece of oil rich land and that the father stood to gain millions if the man were dead, the prosecutor, against sought an indictment from the grand jury. If the grand jury indicts the father the second time, will the father's rights against Double Jeopardy prohibit the prosecution? A. Yes, because the prosecutor is seeking an indictment for the same offense arising from the same facts used to try and get the first indictment. B. Yes, because jeopardy attaches during the indictment proceedings. C. No, because jeopardy had not attached at the first indictment proceeding. D. No, because the prosecutor learned additional facts before seeking the indictment the second time.

C. No, because jeopardy had not attached at the first indictment proceeding. Discussion of correct answer:The Fifth Amendment, applied to the states through the Fourteenth Amendment, protects a person from being prosecuted twice for the same crime. The protection does not attach however unless: (1) the first witness is sworn and evidence is heard in a non-jury trial; or (2) the jury is impaneled and sworn in a jury trial. Neither of these can occur until a grand jury has handed down an indictment. Thus, a prosecutor may seek to indictment a person more than once without being barred by double jeopardy.

A public high school allowed various student groups to meet on school grounds. Each of these groups had secular purpose except for one, which was a Christian group. Students belonging to the Christian group would pray together at their meetings, engage in Bible study, and discuss various current events and social issues from a Christian perspective. A student's parent brought a suit against the school district claiming that the school violated the US Constitution by permitting the Christian group to meet on school grounds. Is the parent likely to prevail? A. Yes, because the Establishment Clause requires a separation of church and state. B. Yes, because religious clubs of other religions do not meet on the school's grounds. C. No, because the group's right to assemble is protected by the Privileges and Immunities Clause of the Fourteenth Amendment. D. No, because prohibiting the club from meeting would be a violation of the First Amendment's Free Speech Clause.

D. No, because prohibiting the club from meeting would be a violation of the First Amendment's Free Speech Clause. Discussion of correct answer:The Supreme Court has held that permitting a religious club to meet on school grounds would not be a violation of the Establishment Clause of the First Amendment [Good News Club v. Milford Central School, 533 U.S. 98 (2001)]. In the same case, the Court ruled that prohibiting religious clubs from meeting on public school grounds violates the Free Speech Clause of the First Amendment.

A man conveyed property via quitclaim deed to his sister. Two months later, his sister was evicted from the property to the man. The sister now brings suit against the man for damages. Should the sister prevail in her action for damages? A. Yes, because the eviction violates the covenant against encumbrances B. Yes, because the eviction violates the covenant of quiet enjoyment C. No, because the deed by which the man conveyed the property to the sister was not a special warranty deed. D. No, because the man conveyed the property to the sister via quitclaim deed.

D. No, because the man conveyed the property to the sister via quitclaim deed. Discussion of correct answer:The sister will not prevail in her action because the man conveyed the property to the sister via quitclaim deed. In a quitclaim deed, the seller does not make any warranties and instead simply conveys whatever interest he has. Given that the deed conveyed by the man was a quitclaim deed, the man is not responsible for the fact that the sister was evicted by the neighbor. Therefore, this answer choice is correct.

A restaurant patron sued a fast-food restaurant, alleging that she slipped and fell on a grease puddle on the floor that the restaurant had negligently failed to clean up within a reasonable time. As part of her case, to show the extent of damages, the restaurant patron offered the following testimony of her boyfriend: When he asked the restaurant patron to go sailing with him the day after the accident, she replied, "Sailing, no way! It will be a. wonder if I can even stand up; my backside is killing me!" Opposing counsel objects to this testimony. Should the trial court rule that the evidence is admissible or inadmissible? A. Inadmissible, because the declarant must be unavailable for a declaration of present bodily condition to be admitted. B. Inadmissible, because a recorded recollection must be in writing. C. Admissible, as an excited utterance. D. Admissible, as a declaration of present bodily condition.

D. Admissible, as a declaration of present bodily condition. Discussion of correct answer:This statement about the restaurant patron's pain is within the hearsay exception for statements of present bodily condition. The present bodily condition exception is separate from the Federal Rule of Evidence exception for statements for the purpose of medical treatment, so it need not be to a medical provider or her agent to be admissible. The declarant's availability is immaterial for both exceptions.

An engineering firm agreed with a car manufacturer to inspect and test a complex parts delivery system used in a new factory that the car manufacturer was constructing. The car manufacturer estimated that this process would take five months to complete. During negotiations, the engineering firm and the car manufacturer agreed that if overtime were necessary, it would be at double time instead of the regular hourly rate. However, the written contract specified only that the car manufacturer was to pay the engineering firm a total of $15,000 in five monthly payments. The agreement further provided that "Any prior agreements are merged herein" and that "This contract may only be amended in writing". After working for 2 months, and after the car manufacturer had paid $6,000 in 2 installments, it was clear to both parties that work on the new system would take more than the five month s allotted. This was because a faculty site survey had not revealed that the soil at the site was unstable. The unstable soil had caused the factory floor to severely subside in 2 places. The car manufacturer indicated to the engineering firm that when work on the factory was able to resume a full speed after repairs were made, it would probably be necessary for the engineering firms people to be on the job for more than 5 months, and most likely for more than 40 hours every week. The car manufacturer agreed orally with the engineering firm to pay them an additional $3,000 a month for every extra month until the job was finished. They also agreed to pay the double-time for all additional hours worked by the engineering firm's people over 40 hours a week. when the project was complete, the engineering firm submitted invoices to the car manufacturer for the fifth month through the eighth month. The car manufacturer had not previously paid the invoices, because the car manufacturers board of directors had told the accounting department not to make any payments until the factory was up and running. The car manufacturer then paid the engineering firm $9,000 for the sixth month through the eighth month. Is it likely that the car manufacturers attempt to exclude the parol evidence will succeed? A. Yes, because the contract clearly states that any amended contract must be written. B. Yes, because the oral agreement alters the written contract. C. No, because the changed situation necessitates the oral agreement and related to a different subject. D. No, because the written contract came first and oral agreement came after it.

D. No, because the written contract came first and the oral agreement came after it. Discussion of correct answer:Generally, parol evidence cannot be admitted if used to contradict the terms of a fully integrated agreement. A completely integrated agreement is evidenced by the terms of an agreement and often includes a "merger" clause. This contract clearly states that prior agreements are merged into the new contract. However, the parol evidence rule only serves to exclude communications and evidence which took place or otherwise arose prior to or contemporaneously with the execution of the written contract. Because the oral agreement in this question occurred after the written contract was executed, the parol evidence rule will not serve to exclude evidence of the oral modification. The subsequent modification will be admitted and enforced.

A cardiologist in State A contracted with a local mechanic to do a complete restoration of his two vintage Italian sports cars. When the cardiologist failed to pay, the mechanic sued. However, since the mechanic moved to State B after the service was completed, he filed the action in State B. The cardiologist's contacts with State B were as follows: (1) on previous occasion, the cardiologist drove through State B to attend a conference there, and paid for a hotel room and other expenses there; and (2) the cardiologist had also considered moving, and he submitted his resume online to some employers in State B. Did the cardiologist implicitly consent to personal jurisdiction in State B? A. Yes, because he operated a motor vehicle in State B. B. Yes, because applying for employment within State B showed an intent to be domiciled there. C. No, because his contacts with State B were minimal. D. No, because this suit does not arise out of his State B contacts.

D. No, because this suit does not arise out of his State B contacts. Discussion of correct answer:Federal courts consider a person to consent to jurisdiction in a state when he or she drives within the state and a cause of action arises from that driving; conducts business within the state; or is present in the state when the conduct that caused the action or dispute took place. However, none of these apply to the cardiologist. The lawsuit concerns the restoration of his sports cars, which was contracted for in State A with a local (at the time) mechanic. Attending a conference in a state is also not a pervasive enough contact to count as "doing business" there. Thus, this answer is correct.

The general manager of an opera company was delighted when he secured a prominent and famous soprano fro the lead role in the upcoming production. Under the terms of the soprano's contract with the opera company, the soprano was to receive a salary of $1,950 per week for the course of the two-month production. The soprano, a highly sought-after performer, turned down an offer from a touring theater company in order to accept the role in the opera company's production. During the second week of the production, however, the soprano's throat grew scratchy and sore, and the doctors diagnosed strep throat. Because the soprano was unable to sing her understudy took over the lead role. By the following week, the soprano was feeling much better and was ready to resume performing for the remaining five and a half weeks of the production. However, the manager refused, stating that he had signed a contract for the understudy to perform the remainder of the show. The soprano filed suit against the manager for breach of contract. Which of the following, if proven, would adversely affect the soprano's likelihood of success in her suit against the manger? A. The soprano declined the manager's offer to be a member of the chorus at a rate of $500 per week. B. The soprano declined the manager's offer to be an understudy to the original understudy at a rate of $1,000 per week. C. By general acclaim, the understudy was more sparkling in the role Thant eh soprano had been. D. The. manager believed that the soprano's recovery period would be longer than the duration of the production.

D. The manager believed that the soprano's recovery period would be longer than the duration of the production. Discussion of correct answer:The doctrine of impossibility excuses both parties from their obligations under a contract if the performance has been rendered impossible by events occurring after the contract was formed. If the manager believed that the soprano's illness and subsequent recovery period would extend through the end of the production, he could properly consider the contract with the soprano discharged due to impossibility. As a result, he would have been free to enter into an agreement with the understudy, and the soprano would not have a breach of contract action against him.

A businesswoman with dual citizenship I the US and UK decided to transfer her entire business overseas. She entered into a contract with a buyer for the sale of her home, which included a large home office, for $325,000. The contract further provided that the transfer of the deed, as well as the buyer's payment of the purchase price, would occur on May 1, one month from the date of the contract. After the signing of the contract, the businesswoman traveled to England to begin the process of setting up her headquarters there. She expected to stay four weeks and to return home the day before the closing date of the contract. However, en route home to the US, the businesswoman found herself stranded in Madrid, Spain, following a layover and several flight delays and cancellations. As a result, she did not arrive home until May 2, the day after the closing date. Upon her return, the businesswoman contacted the buyer to reschedule the closing. However, the buyer was highly offended at the businesswoman's failure to show upon the promised date or to contact him about the delay, and he informed her that he would be cancelling the contract due to her breach. If the businesswoman files an action seeking the buyer's specific performance of the contract, will she prevail? A. No, because unless otherwise provided in the terms of the contract, a party's failure to render performance on the date promised constitutes a repudiation of the contract. B. No, because specific performance is not an appropriate remedy for a buyer's breach of a contract for the sale of land. C. Yes, because the businesswoman's delay in performance was justified under the circumstances. D. Yes, because the contract did not provide that time was of the essence.

D. Yes, because the contract did not provide that time was of the essence. Discussion of correct answer:Generally, a party's failure to perform a contractual obligation by the time specified in the contract is considered only a non-material breach of the agreement. In the event of a non-material breach, the aggrieved party may recover damages caused by the breach, but the aggrieved party is not excused from performing, nor may he properly regard the contract as terminated. In such a case, as long as the other party tenders performance within a reasonable time, the breach is cured, and if the aggrieved party refuses to perform, the other party is entitled to seek specific performance of the contract. However, if a contract specifies that "time is of the essence," a party's late performance is considered a material breach, excusing the aggrieved party from performance. In this case, it does not appear that the parties' contract provided that time was of the essence in the agreement. Therefore, the businesswoman's late performance does not operate to excuse the buyer from performing, and the businesswoman is likely to prevail in her action seeking specific performance. Note that while it is perhaps more common for a buyer to seek specific performance following a seller's breach than for a seller to seek specific performance as a remedy for a buyer's breach, the traditional rule of mutuality of remedy holds that because the buyer is entitled to specific performance for breach of a contract for the sale of real property in the event of the seller's breach, the seller is also entitled to specific performance in the event of the buyer's breach. As such, this answer is correct.

The state was in severe economic depression that threatened to bankrupt the state and cause long-term damage to its industrial sector. In response, the state legislature enacted a statute that required every business with annual sales within the state over $5 million to purchase each year at least $2 million dollars of goods and services produced within the state and to hire at least ten state residents on a full time basis. Which of the following parties most clearly has standing to challenge the constitutionality of this statute in federal court? A. A business in an adjacent state that provides from that adjacent state all of the goods and services bought by a corporation that has annual sales within the state of $8 million. B. A corporation selling $3 million of goods annually in the state, but presently purchasing only $200,000 in goods and services in the state. C. The governor of an adjacent state, on behalf of the state and its residents. D. None of the parties, because the Eleventh Amendment bars suit against the states in federal court.

A. A business in an adjacent state that provides from that adjacent state all of the goods and services bought by a corporation that has annual sales within the state of $8 million. Discussion of correct answer:This business will clearly suffer injury when its purchaser must change to an in-state supplier of goods and services. To have standing in federal court, the claimant must establish a direct injury to the claimant's own constitutional rights, which the federal court can remedy. The injury is clear and directly caused by the state statute because it stands to lose a very large account because the statute requires its client, who has over $5 million in annual sales within the state, to begin purchasing at least $2 million of its requirements in this state instead. This discriminates against interstate commerce, and the federal court can remedy this injury by declaring the law unconstitutional.

A landowner conveyed his property "to A, her heirs and assigns, so long as it is used by the people to gather for purposes of public assembly and debate, then to B and his heirs." Which of the following best describes A's interest in the property? A. A fee simple absolute B. A fee simple determinable C. A fee simple subject to a condition subsequent D. A fee simple subject to an executory interest

B. A fee simple determinable. Discussion of correct answer:The language of conveyance to A creates a fee simple determinable. It contains language of duration—"so long as" or "until" a limiting event or circumstance arises. Once the event or circumstance occurs, the fee automatically reverts to the grantor or his successors. No other language, such as an explicit right of reentry, is required to qualify it as a fee simple determinable. When the limiting event occurs, the property will pass to B, a third party. Unlike a reversion to a grantor, a contingent future interest owned by someone other than the grantor (an executory interest) is subject to the Rule Against Perpetuities. Because B's ownership interest could vest long after a life-in-being plus 21 years, the purported conveyance to him is void. Removing the language creating that gift, a fee simple determinable is created. Thus, A takes a fee simple determinable with a possibility of reverter in the landowner, the grantor.

A businessman owned a tract of land that had a large lake situated on the southeastern corner of the property. The businessman's neighbors children liked to swim in the lake, and the businessman had no objection to their doing so. Eventually, the neighbor approached the businessman to request that the businessman formally grant him an easement allowing him continued access to the lake. The businessman granted the neighbor the easement for a price of $3,500, and the neighbor constructed a paved pedestrian pathway from his own property to the lake. The easement was recorded on the deed to the land. Several years later, the businessman decided to sell the property. He entered into an oral agreement wit his colleague to sell the colleague the property for a price of $4.2 million, to be paid in the form of a down payment of $1 million, followed by monthly installment payments of $5,000. At the time that the parties entered into their agreement, the businessman informed the colleague of the neighbor's easement, and the colleague agreed that the easement would be excepted from the transaction. The colleague then tendered payment of the down payment and moved into the ranch house on the property. He found his new home quite to his liking and made no changes to the house or the surrounding land. Several months later, the businessman decided that he had made an enormous mistake in letting go of his beloved estate, and he told the colleague that he was backing out of their agreement and asked that the colleague vacate the property immediately. The businessman assured the colleague that he would naturally refund the colleague's down payment and the three monthly installment payments the colleague had made. The colleague was horrified and berated himself for not yet recording his deed to the property. If the colleague seeks to enforce the oral agreement for the purchase and sale of the property, is he likely to succeed? A. No, because the colleague did not make any improvements to the land. B. No, because the neighbor's easement rendered the businessman's title unmarketable, thereby voiding the businessman's agreement to sell the colleague the land. C. Yes, because the contract is enforceable under the doctrine of part performance. D. Yes, because the businessman is estopped from denying the existence of the contract.

C. Yes, because the contract is enforceable under the doctrine of part performance. Discussion of correct answer:The Statute of Frauds applies to contracts for the sale of real property. To satisfy the requirements of the Statute of Frauds, a contract for the sale of an interest in real property must be in writing and signed by the party to be charged. The writing must contain: (1) a description of the property; (2) a description of the parties; (3) the price; and (4) any conditions of price or payment, if agreed upon. In certain limited situations, the doctrine of part performance may be used to enforce an otherwise invalid oral contract of sale, provided the acts of part performance unequivocally prove the existence of the contract. For the doctrine of part performance to apply, a showing of some, or all, of the following elements must be made: (a) payment of all or part of the purchase price; (b) taking of possession; or (c) making substantial improvements. In this case, the oral contract between the businessman and the colleague for the sale of the businessman's land would, under most circumstances, be unenforceable. However, given that the colleague tendered payment of part of the purchase price in the form of the $1 million down payment and three installment payments, and that the colleague had taken possession of the property and lived there for several months, it is likely that the parties' oral agreement will be upheld under the doctrine of part performance, despite the lack of an evidencing writing. As such, this answer is correct.

For over 20 years, a husband had been saving money to renovate an abandoned building for his wife to use as an art studio. After finally saving up enough money, the husband contracted in writing with a builder to renovate the building. According to the contract, the husband agreed to pay the builder $15,000 to perform the renovatition work. The agreement recited that the husband was hiring the builder to remodel the office as a present for his wife. Relying on her husband's contract, the wife, who had been saving money of her own and had an option to lease a different building, allowed the option to expire and told her husband and the builder. After beginning the renovation work, the builder discovered there was a potentially hazardous chemical compound throughout the walls and ceilings. As a result, the builder notified the husband that in order to renovate according to the plans, the chemical compound would have tore removed at an additional cost of $25,000. Neither the husband nor the builder had reason to anticipate that the chemical compound would prevent the builder from performing the remodeling work. Upon discovery of the chemical compound, however, the husband and the builder agreed to radically alter the renovation work so that the chemical compound did not have to be removed. The wife did not learn of the changes until after the builder completed the remodeling. If the wife finds the changes to be unacceptable, will she succeed in an action against the builder for not renovating according to the original plans and specifications? A. No, because the wife was only an incidental beneficiary under the terms of her husband's contract with the builder. B. No, because the wife's rights under her husband's contract with the builder were subject to any defenses that the builder could have raised against the husband. C. Yes, because the wife was an incidental beneficiary and her rights properly vested when she assented to the contract. D. Yes, because the wife changed her position in reliance on the original contract that was entered into between her husband and the builder.

D. Yes, because the wife changed her position in reliance on the original contract that was entered into between her husband and the builder. Discussion of correct answer:This question deals with the vesting of a donee beneficiary's rights. If the promisee intends the promised performance as a gift to the third party, the third party is a donee beneficiary. Such intention is clear where performance of the promise will result solely in benefit to the third party and not to the promisee. Here, the builder's performance of the husband's promise was intended as a gift to benefit the wife, not the husband. Thus, the wife is a donee beneficiary. Under the majority view, the rights of a donee beneficiary, like those of a creditor beneficiary, vest when the third party with knowledge assents to the promise. Before the time such rights are vested, a beneficiary cannot prevent modification or rescission of the contract by the promisee and the promisor. In this case, however, the wife's rights had vested before her husband and the builder agreed to alter the renovation work. By allowing the lease option for another building to expire and notifying her husband and the contractor of this fact, the wife changed her position in reliance on her husband's gift, and her assent to the contract is therefore presumed. As such, the wife can enforce the original terms of the contract.

A woman sued a man in federal court for intentional infliction of emotional distress after he pulled an inappropriate prank on her at this workplace. The man told his attorney that he bought the snake that he placed on her chair that day at a pet store. He quit his job shortly after the incident. The woman sent interrogatories to he man's attorney asking whether he had bought a snake on the day of the incident. Does the attorney have an answer the interrogatories? A. Yes, because they involve questions of fact. B. Yes, because the man quit his job shortly after the incident. C. No, because the statements are protected by the attorney-client privilege. D. No, because the statements are protected by the work product doctrine.

A. Yes, because they involve questions of fact. Discussion of correct answer:The protection of privilege belongs only to communications between an attorney and his or her client, not facts. Consequently, while a client cannot be compelled to answer questions regarding what he or she communicated to his or her attorney, the client cannot create a privilege simply by telling something to his or her attorney. Here, the interrogatories are asking about the facts themselves, not whether they were communicated to the attorney. Additionally, it would be an ethical violation for the attorney for his client to commit perjury by filing answers the attorney knew to be untrue.

A fight broke out between a bookkeeper and a technician while the two were watching a roller derby match, and the technician died as a result of his injuries. The bookkeeper was charged with murder. At his criminal trial, the bookkeeper called a witness to testify that he had seen the fight and that from what he had observed, the technical had been the aggressor. The witness also testified that he had worked with the bookkeeper and the technician for several years and, in his opinion, the bookkeeper was a peaceful man, but the technical had a reputation for violence. The prosecution then called on the technicians son to the stand, who proposed to testify that his father was a peaceful, non-violent person. The defense objected. Is the son's testimony admissible? A. No, because the bookkeeper has not specifically questioned the technician's character. B. No, because the technicians character may not be established by the testimony of his son. C. Yes, to rebut the witness's testimony that the technical had a reputation for violence. D. Yes, to rebut the bookkeeper's claim of self-defense.

C. Yes, to rebut the witness's testimony that the technician had a reputation for violence. Discussion of correct answer:When a claim of self-defense is made, evidence of the character of the deceased for violence may be introduced by the defendant. Once this is done, the door is open for the prosecution to offer contrary evidence to rebut. Additionally, under FRE 404(a)(2)(C), the testimony that the victim was the initial aggressor is also a trigger that allows evidence in rebuttal. The testimony here is proper rebuttal because the defense "opened the door" with the witness's testimony that the technician had a reputation for violence.

Congress passed a law creating the Federal Pet Food Agency, which empowered to oversee the safety and nutritional value of pet food sold in the US. The statute also empowered the agency to issue "cease and desist" orders to pet food manufacturers who flagrantly or intentionally flouted the regulations promulgated by the agency. Violation of the cease and desist orders were made punishable by fine or imprisonment. Finally, the statue provided that the president would nominate five members to the agency, the Senate would nominate three members, and the Speaker of the House would nominate two members. What is the strongest argument against the constitutionality of the statute? A. Congress may not usurp the constitutional authority of the president to nominate officers to an agency with administrative powers. B. The statute failed to state whether the members of the agency had to be confirmed by the Senate. C. The statute failed to provide for a removal process for members of the agency. D. The members of the agency were inferior officers, and, as such, Congress did not have the authority to delegate the appointment power to the president.

A. Congress may not usurp the constitutional authority of the president to nominate officers to an agency with administrative powers. Discussion of correct answer:Article II, Section 2 of the U.S. Constitution specifies that the president shall appoint "with the advice and consent of the Senate," all "Ambassadors, other public Ministers and Consuls, Judges of the Supreme Court, and all other Officers of the United States." Although Congress may appoint officials to exercise such investigative power as it might delegate to one of its own committees, it may not appoint members of an agency or commission with administrative powers. Such persons are deemed "officers of the United States" and must be appointed by the president pursuant to Article II, Section 2 [Buckley v. Valeo, 424 U.S. 1 (1976)].

A soccer mom was arrested after striking another parent in the face with her purse. The blow was sufficiently forceful to break the victim's nose. At her trial on assault and battery charges, the soccer mom claimed that she struck the other woman in self-defense to fend off her attack. The soccer mom further alleged that the other parent's attack was motivated by the news that the soccer mom's daughter had edged out of the other parent's daughter for a spot on the high school soccer team. At trial, the soccer mom wished to call the president of the PtA to testify that the soccer mom was a good parent who will go the extra mile to ensure the welfare of the students of the school. How will the court rule with respect to the admissibility of the PTA president's testimony? A. Her testimony is inadmissible, because the testimony does not address a pertinent character trait. B. Her testimony is inadmissible, because in the form of an opinion is not a proper method of proving character. C. Her testimony is admissible, because the defendant in a criminal trial is permitted to introduce opinion evidence of her good character. D. Her testimony is admissible, because testimony in the form of an opinion is a proper method of proving character.

A. Her testimony is inadmissible, because the testimony does not address a pertinent character trait. Discussion of correct answer:Under Federal Rule of Evidence 404, "evidence of a person's character or a trait of his character is not admissible for the purpose of proving that he acted in conformity therewith on a particular occasion," although the accused in a criminal trial may offer reputation or opinion evidence of a pertinent trait of his good character to prove that he is innocent of the offense charged. For example, where, as under the facts presented, a defendant is charged with assault and battery, the accused may offer evidence of her character for peacefulness. In contrast, evidence of a defendant's propensity for being a good parent is not pertinent to a charge of assault and battery. Thus, under the facts presented, this is the best answer. The PTA president's testimony will be ruled inadmissible, because it does not address a pertinent character trait.

A lawyer was angered after he was unexpectedly laid off from his longtime job with a prestigious firm. The next day, he returned to the firm and fired shotgun rounds into the air. The lawyer later testified, without contradiction, that he had not intended to kill anyone, but simply sought to exact revenge on the firm's managing partner. The lawyer's plan was to scare everyone so that they would leave work for the day, causing the firm to miss a critical deadline on a case the lawyer had been working on. Unfortunately, one of the bullets ricocheted off the wall and killed the lawyer's former secretary. The crimes below are listed in descending order of seriousness. On these facts, what is the most serious offense for which the lawyer could properly be convicted? A. Murder. B. Voluntary manslaughter. C. Involuntary manslaughter. D. Assault.

A. Murder. Discussion of correct answer:The man could properly be convicted of murder, even though he lacked a specific intent to kill, because his conduct created such a high risk of death and was so devoid of social utility that he could be found to have acted with a depraved heart.

On January 1, a seller received an order in the mail from a buyer requesting shipment of its latest computer at the seller's current catalog price. The order specified that the seller could accept the offer either by shipping the goods or promptly notifying the buyer of acceptance. On January 2, the seller mailed to the buyer a letter accepting the order and assuring the buyer that the computer would be shipped on January 3. On January 3, the seller realized that he was out of the computer model, and instead shipped to the buyer a different computer model and a notice of accommodation. On January 5, the buyer received the seller's January 2 letter as well as the different computer model, but not the notice of accommodation. Which of the following best states the parties' legal rights and duties? A. The buyer can either accept or reject the different computer model, and in either event, can recover damages, if any, for breach of contract. B. The buyer can either accept or reject the different computer model, but if he rejects it, he will thereby waive any remedy for breach of contract. C. The seller's prompt shipment of nonconforming goods constituted an acceptance of the buyer's offer, thereby creating a contract for sale of the replacement computer model. D. The seller's notice of accommodation was timely mailed, and his shipment of the different computer model constituted a counteroffer.

A. The buyer can either accept or reject the different computer model, and in either event, can recover damages, if any, for breach of contract. Discussion of correct answer:Section 2-206(1)(b) of the Uniform Commercial Code (UCC) provides that a seller's shipment of nonconforming goods with a notice of accommodation does not constitute an acceptance and breach, but rather, constitutes a counteroffer, which the buyer is free to either accept or reject. This section further provides, however, that a contract calling for prompt shipment can be accepted by either a prompt promise to ship or the prompt shipment of goods. However, absent accommodation, the seller's shipment of nonconforming goods constitutes an acceptance of the buyer's offer under the UCC. It also constitutes a breach of the resulting contract. Here, the seller accepted the buyer's offer by a promise to ship when he mailed his letter on January 2, but the buyer did not receive the notice of accommodation at the time. Under the UCC, a buyer may accept or reject nonconforming goods and, in either event, may recover damages. Given that the seller shipped the goods on January 3, but failed to conform to the contract (formed on January 2 when he mailed his letter of acceptance), the buyer has a valid cause of action for breach of contract.

A ski instructor purchased a new brand of sunscreen made especially for sensitive skin on a person's face and neck. He took the unopened product to his first ski class on the day and offered it to one of his students, who had forgotten her own sunscreen. When the student opened the sunscreen, all of the skiers in her vicinity noticed a very strong chemical odor. The student didn't believe that the smell came from the product, though, and applied the sunscreen to her face and neck. Within minutes, a painful rash had broken out on the student's face. It was later determined the manufacturer of the sunscreen had inadvertently included a harsh cleaning agent in the batch of sunscreen used by the student. The student filed a strict products liability claim agains the manufacturer. The manufacturer claims that the student should not recover because she assumed the risk by applying sunscreen with such a strong chemical order. Is the manufacture likely to succeed in asserting this defense? A. No, because the student did not comprehend the risk. B. No, because a plaintiff's conduct cannot negate or reduce the plaintiff's strict product liability claim. C. Yes, because the student could smell the chemical odor and exposed herself to it nonetheless. D. Yes, because the student assumed the risk by applying the sunscreen.

A. No, because the student did not comprehend the risk. Discussion of correct answer:In strict products liability, the general rule is that the failure of the plaintiff to discover a defect that seems obvious is not a valid defense. A court will only find that the plaintiff assumed the risk of using the defective product if the plaintiff: (1) knew of the defect; (2) comprehended the risks; and (3) willingly exposed himself to the risk. In this case, the student smelled the chemical odor, but didn't believe that the odor came from the sunscreen. Therefore, even if every other skier in the class would have realized that something was wrong with the sunscreen, the fact that the student herself did not comprehend the risk of using the sunscreen will prevent the manufacturer from succeeding in his argument that the student assumed the risk of harm.

A state legislature passed the "Health and Safety I Marriage Testing and Education Act", which required all persons applying for a marriage license to undergo screening for Human Pailloma Virus (Sexually transmitted disease that is frequently asymptomatic and can lead to cervical cancer) and for Hepatitis C (a communicable disease that can lead to liver failure). Under the Act, all applicants for a license were also required to attend a Saturday class to learn how to prevent the transmission of these diseases from one sexual partner to the other. The purpose of the Act was to provide prospective spouses with information about two harmful diseases that have severe health consequences and to slow the rate of the spread of these diseases in the state's population. A man who lives in this state intends to propose to a woman who also resides in this state. However, the man does not wish to fulfill the requirements of this Act necessary to obtain a marriage license. If the man challenges the Act in federal court on constitutional grounds, which of the following offers the strongest grounds on which the state can argue for the suit's dismissal? A. Ripeness. B. The man and woman are both state residents. C. The suit does not present a federal question. D. The federal court cannot hear a political question.

A. Ripeness. Discussion of correct answer:To satisfy the "case or controversy" requirement of Article III of the federal constitution, a plaintiff must first establish standing to sue by showing a direct injury in fact, economic, or otherwise. In order to satisfy the "ripeness" requirement, a plaintiff must experience an actual injury or a threat of imminent injury. Unless it is certain that an actual "controversy" exists between the parties, the federal court will not accept a case for review. Here, the man has not yet suffered a direct injury, nor is he under an imminent threat of injury in fact, in his economic situation or otherwise, because he has not yet applied for a marriage license or been refused such a license. In fact, the man is not even engaged. As such, the man does not have sufficient standing to bring his claim, and the lack of ripeness is the state's strongest argument for dismissal of his lawsuit.

A landowner owned a parcel of land containing about 90 acres. It included a small house on the property. He decided to build a newer, larger house on a different section of the property. He agreed to sell the house to a co-worker. He signed an agreement with her to sell her the house and land around the house that would be "sufficient to build a lawn and a vegetable garden", both of which the co-worker wanted. The sales price was $45,000, and the co-worker paid the landowner $100 on account. The co-worker then failed to perform her part of the agreement. The landowner sued her for specific performance. The court would rule in favor of which party? A. The co-worker, because the agreement is very ambiguous. B. The co-worker, because her $100 payment will not support a contract, being such a nominal sum for consideration.. C. The landowner, because the written agreement satisfies the Statute of Frauds. D. The landowner, because he had a written agreement with the co-worker, who was the party to be charged therewith.

A. The co-worker, because the agreement is very ambiguous. Discussion of correct answer:The suit is based on specific performance. The phrase "sufficient land for a lawn and vegetable garden" is too vague and ambiguous. A court will enforce a breached contract on the theory of specific performance. However, the court must have an idea of what it is to specifically enforce, so the terms of the agreement must be definite and certain. When an order for specific performance is made, the party in the wrong must be told exactly what to do to comply with the court's order. The landowner is trying to force the co-worker to pay, but in the agreement there was no indication of how much land the co-worker needed or wanted for a lawn or for her vegetable garden. Lacking such a definite and certain term as the amount of land in question, the landowner would fail. The court will not force the co-worker to perform until it is clear how much land she is purchasing.

A man was found shot in his apartment. The police arrested one of the victim's friends. During its case-in-chief, the prosecution offers into evidence a sketch, to be authenticated by a police officer who will testify that he found the victim lying bleeding on the floor of his living room and asked who had done this to him, but that the victim could not answer, because he was coughing up blood. The officer will further testify that the victim grabbed his pen and paper trash bag from the floor and drew a picture of a face with a crooked nose and a jagged line on the left side. The victim then died in the hospital. The prosecution asserts that the defendant's nose had been broken several times and was noticeably crooked as a result, and that the defendant had a prominent scar on his left cheek. The prosecution authenticated the offer of proof and the defendant objected to the offer. How should the court rule? A. The court should admit the sketch as a dying declaration. B. The court should admit the sketch as a recorded recollection. C. The court should not admit the sketch, because it is inadmissible opinion evidence. D. The court should not admit the sketch, because it is hearsay not within any exception.

A. The court should admit the sketch as a dying declaration. Discussion of correct answer:Federal Rule of Evidence 801(a) provides that a "statement" is: (1) an oral or written assertion; or (2) nonverbal conduct of a person, if it is intended by the person as an assertion. Therefore, the victim's drawing is nonverbal conduct that could be considered a "statement." Pursuant to Federal Rule of Evidence 804(b)(2), a hearsay statement made by the declarant "while believing that his death was imminent, concerning the cause or circumstances of what he believed to be his impending death," is admissible as an exception to the rule excluding hearsay evidence. Here, because the victim was coughing up blood, he could easily have believed that he was about to die and, in fact, died shortly thereafter. In addition, his sketch related to the circumstances and cause of his death, in that the sketch identified his killer. Therefore, the sketch and the officer's authenticating testimony are admissible as a dying declaration.

The defendant is charged with shooting and killing a victim with the defendant's .357 Magnum revolver. According to evidence obtained by police investigating the case, the victim showed up drunk at the defendant's apartment. When the defendant opened the front door, the victim began yelling at the defendant. The defendant was unable to exit the apartment, because the victim was blocking the door, and there was no alternate exit route. When the victim began yelling even more loudly and threateningly, the defendant drew his gun and shot the victim from the back corner of the living room. The bullet struck the victim in the jugular vein. The defendant called for medical assistance, but the victim bled to death before the ambulance arrived. The defendant was charged with murder and has asserted self-defense. Upon commencement of his trial, the defendant immediately attempts to call to the stand various witnesses to testify to the victim's reputation as. a belligerent and intimidating man who regularly terrorized the neighborhood. The defendant also presents testimony including specific instances in which the victim terrorized multiple neighbors, including the defendant. Prior to the witness' testimony, the state moves to bar admission of all the testimony concerning the victim's character and reputation. How should the judge rule? A. The evidence is admissible, because it is relevant to the defendant's state of mind, and, therefore, to his self-defense claim. B. The evidence is inadmissible, because the character evidence cannot be used to show that a party acted in conformance therewith on a specific occasion. C. The evidence is inadmissible, unless the testifying witnesses were present for all the events. D. The evidence is inadmissible, unless the victim is not available to defend his reputation.

A. The evidence is admissible, because it is relevant to the defendant's state of mind, and, therefore, to his self-defense claim. Discussion of correct answer:While the introduction of character evidence to prove that a party acted in conformance therewith on a particular occasion is generally prohibited under Federal Rule of Evidence 404, such evidence may be admissible under certain particular circumstances, as where an accused seeks to introduce pertinent evidence of the character of the victim in support of a claim of self-defense to a charge of homicide. Here, the defendant was charged with the victim's homicide, and evidence of the victim's reputation for violence is relevant to the validity of the defendant's state of mind and his claim of self-defense. As such, the evidence is admissible.

A city received $4 million in federal funding to construct an inner-city athletic facility. An appropriate site was chosen for the facility, and the city council earmarked $3 million for the project. At the same time, the city council set aside the remaining $1 million of the grant to enhance the waterfront property for use by the general public by adding walkways, bikeways, a small park, and a retail facility providing sports equipment rentals. The funds from the commercial site are to be channeled back into the inner city recreational site. The athletic facility site was one mile inland from the waterfront property, which was accessible from the inner city by a system of subways and sidewalks. The city council, in making its allocation of the federal funding, determined that its plan promoted the health and recreation of the general public. The federal government disputed the allocation of funds and had the funds frozen after arguing in federal court that the plan violated the terms of the grant. The city filed a motion seeking release of the funds so that the plan could be implemented as conceived. What is the likely outcome for the city's motion? A. The federal government will prevail, because the federal government has the power to condition receipt of the funds on the recipient's conformance to federal regulations. B. The federal government will prevail, because the doctrine of state sovereignty is inapplicable to an action between a local governmental body and the federal government. C. The city will prevail because the city's plan for construction of recreational facilities substantially complies with the terms of the federal grant. D. The city will prevail, because federal intervention in implementation of the city's recreational facilities violates states sovereignty.

A. The federal government will prevail, because the federal government has the power to condition receipt of the funds on the recipient's conformance to federal regulations. Discussion of correct answer:Under Article I, Section 8 of the Constitution, Congress is authorized to spend for the general welfare. When a state recipient accepts federal money, the recipient, whether a statewide or local governmental body, is required to spend the money according to specifications set by the federal government. Under the spending power, Congress may require "voluntary" conformance to federal regulations, provided that the conditions attached to receipt of the funds are related to the goals of the spending program. Federal power to condition receipt, therefore, is not absolute. The condition that the funds granted to the city be spent on athletic facilities within the inner city is related to the federal goal of promoting the health of a certain constituency within the city, and the spending program will be upheld.

One evening a fireman went bowling at a local bowling alley. After he had finished bowling, he picked up and put now hat he believed to be his shoes, and went home. About one hour later, as the fireman got ready for bed, he discovered his mistake. Since it was nearly midnight and the bowling alley was already closed, the fireman decided to wait until the next day to return a serious automobile accident. The shoes, which turned out to be quite valuable, were destroyed. The actual owner of the shoes, a handyman, then asserted a claim for conversion against the fireman for the value of the shoes. Who will prevail? A. The handyman, because the fireman intended to take the shoes and they were destroyed while in his possession. B. The handyman, because the fireman committed trespass to a chattel by failing to promptly return the shoes. C. The fireman, because he honestly believed the shoes to be his own. D. The fireman, because he tried to return the shoes within a reasonable time after discovering his mistake.

A. The handyman, because the fireman intended to take the shoes and they were destroyed while in his possession. Discussion of correct answer:The handyman is likely to prevail in a claim of conversion. Conversion is an intentional act that causes the destruction of (or a serious and substantial interference with) chattel. Mistake is not a defense to conversion; the defendant may be held liable regardless of whether he realized the significance of his act. Here, the fireman intended to take the shoes (not realizing, of course, that they were not his). Therefore, he is likely liable for conversion.

A woman had lived in Apartment 123 in the same apartment complex for 6 years. On many prior occasions, the woman had hired the same handyman to perform odd jobs in her apartment. The woman, who was leaving town on vacation, telephoned the handyman and said, "if you will replace the tile in my bathroom while I'm on vacation, I will pay you $700. You can get the keys from the superintendent." The handyman responded, "It's a deal." During their telephoned conversation, the woman failed to tell the handyman that she no longer lived in Apartment 123, but had moved down the hall into Apartment 132. The handyman came to the apartment complex, got the key for Apartment 123 from the superintendent, and re-tiled the bathroom. When the woman returned from her vacation, she noticed her bathroom tile had not been replaced. She contacted the handyman, who then informed her that he had replaced the tile in Apartment 123. The handyman demanded that the woman pay him $700, but the woman refused. If the handyman sues the woman for payment of the $700 and the woman claims mistake, for whom the court award judgment? A. The handyman, because the woman should have realized that he would replace the tile in Apartment 123. B. The handyman, because even though no contract existed, he is entitled to quasi-contractual relief under the circumstances. C. The woman, because the handyman did not replace the title in the woman's bathroom. D. The woman, because no contract existed due to the mutual mistake of the parties.

A. The handyman, because the woman should have realized that he would replace the tile in Apartment 123. Discussion of correct answer:As a general rule, a unilateral mistake will not prevent the formation of a contract. In such cases, the mistake must be substantial. In this case, there is no question that a contract was formed between the woman and the handyman. The woman hired the handyman to re-tile her bathroom, but she failed to inform him that she had moved into a different apartment. Certainly, the handyman's mistake was substantial, in that he re-tiled the bathroom in the wrong apartment. However, because the handyman's mistake was due to the woman's own failure to inform the handyman that she had moved to a new apartment, the woman will not be allowed to avoid the contract.

A homeowner obtained and moved onto a large plot of land, occupying a significant portion of the property. The homeowner lived not hat portion of the land for more than 22 years, believing that the whole plot was his own. Recently, however, the homeowner learned that the deed he had acquired was in fact defective. Now, a person with a valid deed is claiming the entire plot of land for himself. The statutory period for adverse possession in this jurisdiction is 20 years. What rights, if any, does the homeowner have with respect to the plot of land? A. The homeowner owns the whole plot of land, because he acquired title to the land by adverse possession. B. The homeowner owns the portion of the property on which he lived, but not the whole plot because he did not possess the whole plot. C. The homeowner does not own any part of the land, because his possession was not hostile. D. The homeowner does not own any part of the land, because he did not possess the whole plot of the land.

A. The homeowner owns the whole plot of land, because he acquired title to the land by adverse possession. Discussion of correct answer:Adverse possession requires proof of possession that is: (1) open, visible, and notorious; (2) actual; (3) exclusive; (4) hostile and under a claim of title or right; and (5) continuous for the statutory period. A person actually possesses property when he is physically on the premises. A person may also possess the premises constructively. Constructive possession requires color of title (usually a defective deed) and actual possession of at least a significant part of the premises. If those elements are satisfied, the possessor constructively possesses the whole property, as described in the deed. Here, all of the elements of constructive adverse possession are satisfied. The homeowner possessed the land under color of title (a defective deed), and he actually possessed a significant portion of the premises. Finally, the homeowner's possession of the land was continuous for more than 20 years. As such, he now owns the whole plot of land.

A woman entered into a written lease to rent a condo for a term of eight months at a rental of 41,000 to be payable on the first day of each month. The woman's lease ended on July 31. During the leasehold period, the woman fully complied with the terms of the lease and made her monthly rental payments on schedule. When the lease ended at the end of July, the woman decided to stay on and continue to occupy the condo. However, the landlord did not sign a new lease with the woman. At the end of November, the landlord found a new tenant who offered to pay $1,200 per month for the condo. The landlord agreed to show the tenant the condo prior to his move-in date, so he could measure the space for the furniture. However, when the landlord entered the condo, he noticed that the woman was still residing there. The landlord is entitled to which of the following remedies? A. The landlord may recover possession of the condo and the market value of the rent per month for August, September, October and November. B. The landlord may recover possession of the condo and $1,000 rent per month for August, September, October and November. C. The landlord may recover possession of the condo and $1,200 rent per month for August, September, October and November D. The landlord may recover possession of the condo and $2,000 rent per month for August, September, October and November

A. The landlord may recover possession of the condo and the market value of the rent per month for August, September, October and November. Discussion of correct answer:By remaining in possession of the leased premises after the end of the lease term ("holding over"), the woman became a tenant at sufferance. In this situation landlord may recover possession as well as damages. Recovery is not for rent as such (because a tenancy at sufferance is not a true tenancy at all), but for the market value of the rent for the relevant time period. This may be greater than the rent payable during the original term. Therefore, the landlord may recover possession as well as the market value for the hold-over period. Thus, this is the correct answer choice.

A man approached his neighbor, who was seven months pregnant, and said, "If you name your son after my father, I will pay you $200 per month for three years." The neighbor agreed and named her child after the man's father. Afterwards, the man reneged on his promise, and told his neighbor that he had decided not to pay her the money. The neighbor sued the man for breach of contract. For whom should a court award judgment? A. The neighbor, because there was a bargained-for exchange. B. The neighbor, because her promise to name the child after the man's father was a condition of a gift made in consideration of carrying out a moral obligation. C. The man, because his promise to pay the neighbor was a mere gift unsupported by consideration. D. The man, because the neighbor did not experience any recognizable detriment in the naming of her child.

A. The neighbor, because there was a bargained-for exchange. Discussion of correct answer:Consideration must consist of a benefit to the promisor or a detriment to the promisee. Ordinarily, courts do not inquire into the adequacy of consideration, particularly when the values exchanged are uncertain or difficult to measure. In this case, the neighbor's promise to name her child after the man's father was the product of a bargained-for exchange that was supported by consideration. Therefore, the neighbor will prevail in the breach of contract action.

After a rash of accidents, including several fatalities, the. City Council enacts an ordinance prohibiting messenger bicycles from a five-block area in midtown during the morning and evening commute. The owner of a messenger service company that serves many business and law firms in the city and employee 17 messengers files suit in federal court attacking the constitutionality of the ordinance. If the city attorney files a motion to dismiss the owner's suit claiming a lack of standing and the court denies the motion, what is the most likely reason? A. The owner is treated with economic harm. B. The messenger employed by the owner will suffer economic injury. C. The ordinance violates the privileges and immunities guaranteed by the Fourteenth Amendment. D. The ordinance infringes upon the fundamental right to work.

A. The owner is threatened with economic harm. Discussion of correct answer:Article III of the U.S. Constitution requires that a person litigating a constitutional question must show: 1) injury in fact; and 2) causation (i.e., proof that the relief sought will redress the harm or injury that is alleged). The owner has standing because he can show a direct injury or imminent threat of injury to his economic interest in the messenger service. The ordinance prohibiting messenger bicycles from midtown streets during morning and evening commuting hours prevents the owner's 17 employees from performing their job for a substantial portion of the working day. This infringement on their working hours will have a substantial economic effect on the owner's business income. Thus, the owner can assert an imminent threat of economic injury to himself caused by the ordinance, and that a finding that the ordinance is unconstitutional will remedy the harm. Therefore, the owner satisfies the standing requirement.

The owner of 25 lots a residential subdivision telegraphed a buyer on September 15. The owner's telegram indicated that she would sell the buyer any or all of the lots in the residential subdivision of $10,000 each. She also indicated that more details would be sent by mail. The owner followed this up with a letter to the buyer with the necessary details regarding mortgages, terms of payment, and insurance. According to the owner's letter, the offer would remain open until October 15. On September 17, the buyer telegraphed his acceptance of the offer to the owner. He indicated he wanted to buy lot 12, and a corner lot on the main road. The owner and the buyer then entered into a valid contract for the sale of lot 12. The buyer and the owner both knew that the residential subdivision comprised 25 lots, numbered 1 through 25. Assume that the buyer telegraphed the owner on September 21 that he would buy the rest of the lots. On September 25, the owner discovered that her title to the remaining lots was not good. With regard to the sale of the remaining 24 lots, which of the following would best support the owner's contention that she is not liable for a breach of contract if the buyer sued her? A. The owner's initial telegram did not constitute an offer, because it did not specify the specific property to be sold. B. Excuse by failure of an implied condition precedent. C. Unilateral mistake as to basic assumption. D. Impossibility of Performance.

A. The owner's initial telegram did not constitute an offer, because it did not specify the specific property to be sold. Discussion of correct answer:This is the correct answer because the owner sent a telegram not specifying the specific property to be sold. Instead, she told the buyer that he could buy any or all of the lots. The buyer's telegram of September 17 could be construed as the effective offer, and the owner's acceptance to buy lot 12 occurred when the two entered a valid contract for the sale of the real estate. Thus, the initial telegram was not a valid offer, nor did the buyer do anything to re-enforce the offer for the rest of the lots, and so the owner would have no obligation to the buyer as to the rest of the lots.

An uncle wanted his nephew to quit smoking, because smoking is unhealthy. The uncle attempted to convince the nephew to quit, but the nephew would not agree. The uncle then offered to pay the nephew $5,000 if the nephew quit smoking for one year. It was the uncle's hope that if he could get the nephew to stop smoking for one year, he would not pick. up the unhealthy habit again. The nephew agreed, and signed a contract to that effect with the uncle. The nephew quit smoking for one year, but afterward, restarted smoking. He asked for the $5,000, but the uncle refused to pay. The nephew has now filed suit against the uncle. What is the likely outcome of the case? A. The uncle is liable, because there was a bargained for exchange. B. The uncle is liable, because the nephew relied to his detriment on the uncle's promise. C. The uncle is not liable, because the consideration was inadequate. D. The uncle is not liable, because the uncle did not receive a benefit and the nephew did not suffer a detriment.

A. The uncle is liable, because there was a bargained-for exchange. Discussion of correct answer:The majority of jurisdictions evaluate consideration based on the bargained-for exchange inquiry. A minority of jurisdictions use a benefit-detriment analysis instead. Whether courts are applying the benefit-detriment test or the hybrid "promise in exchange for a detriment" test, the question of whether a particular performance by the promisor constitutes a detriment is answered by the application of the so-called legal detriment test. Under this test, the question is not whether the promisee will actually suffer or end up with some sort of net loss due to the transaction. Rather, the question is whether the promisee is doing something he had a legal right not to do or forgoing some activity in which he had a legal right to engage. Here, the nephew avoided an activity in which he had a legal right to engage—namely, smoking. Therefore, there was consideration to support this contract, and the uncle is liable.

A principal of a public high school refused to allow a student to attend the school on the grounds that the student was an undocumented immigrant. The student brought a constitutional challenge against the school district. Is the student likely to prevail? A. Yes, because discriminating against immigrant children does not further a substantial goal of the state. B. Yes, because the law would fail a strict scrutiny review. C. No, because undocumented immigrants are not a suspect class. D. No, because the Fourteenth Amendment applies only to citizens of the US.

A. Yes, because discriminating against immigrant children does not further a substantial goal of the state. Discussion of correct answer:Because undocumented immigrants are not a suspect class, discrimination against them by state actors is subject to rational basis scrutiny. However, in Plyler v. Doe [457 U.S. 212 (1982)], the Court held that preventing the children of undocumented immigrants ("illegal aliens," at the time of the decision) from accessing free public elementary and secondary education failed rational basis review because such discrimination could "hardly be considered rational unless it furthers some substantial goal of the state." In doing so, the Court used a stringent version of the rational basis standard--in essence, it applied intermediate scrutiny. Thus, while undocumented immigrants are not a suspect class and a law discriminating against them would be subject to rational review, a law discriminating against the children of undocumented immigrants would be subject to a higher scrutiny standard. Because the law in question here would not further a substantial goal of the state, it fails the standard set forth by the Supreme Court in Plyler, making this answer choice correct.

A driver was arrested and charged with involuntary manslaughter after a head on collision that killed the other driver. The prosecution claimed that the driver fell asleep while driving and drove into oncoming traffic. At trial, the prosecution called a witness to testify. The witness testified that she was visiting with the driver the morning of the collision when the driver told her he had not gotten any sleep the night before because of a terrible migraine headache. The driver objected to the witness's testimony. Should the court admit the testimony ? A. Yes, because it is non hearsay. B. Yes, because it is a statement of mental or physical condition. C. No, because it is more prejudicial than probative. D. No, because it was not against the driver's interest when he said it.

A. Yes, because it is nonhearsay. Discussion of correct answer:The prosecution is offering the testimony of the witness as to a statement made by the driver, a party-opponent. Statements of party-opponents are admissible and are treated as non-hearsay under Federal Rule of Evidence 801(d).

A convicted murderer was in prison fulfilling a life sentence. Police believed that in addition to the murder he was convicted of, the murderer was also responsible for three other unsolved murders in the area. The unsolved murders had factual circumstances similar to those surrounding the case for which the man was convicted. An undercover officer was sent to jail to pose as an inmate and try to get a confession from the man. The undercover agent posed several questions to the man that the agent believed would et the man talking about the crimes. The undercover officer did not give the man Miranda warnings, and the man ultimately confessed. Will the confession be admissible if the man is tried for the other three murders? A. Yes, because Miranda warnings were not necessary. B. Yes, because the statements were voluntary. C. No, because the man was in custody. D. No, because the man was being interrogated.

A. Yes, because the Miranda warning was not necessary. Discussion of correct answer:The purpose of the Miranda requirements is to protect the due process rights of people accused or suspected of crimes. In other words, Miranda tries to protect people from feeling like they have to talk to police in certain situations. This fact pattern is based off of Illinois v. Perkins [496 U.S. 292 (1990)], in which the Supreme Court held that Miranda is not triggered when an inmate believes he is talking to another inmate. In that situation, the defendant does not need the Miranda warnings because he doesn't even know he is talking to an officer and can therefore not feel that he has no choice but to talk because of the officer's position of authority. Miranda protections only occur when there is an "official" interrogation

An activist was injured during an anti-war demonstration when a police officer hit him with a truncheon because he would not disperse. The activist sued the police department for battery and violation of his constitutional rights. Both the activist and the police department were citizens of State A, and the activist claimed $25,000 in damages. Can the suit be heard in federal court? A. Yes, because the battery and civil rights violation were part of the same occurrence. B. Yes, because the constitutional rights claim involves a federal question. C. No, because battery is a state law claim. D. No, because the suit does not meet the requirements of diversity jurisdiction.

A. Yes, because the battery and civil rights violation were part of the same occurrence. Discussion of correct answer:Supplemental jurisdiction allows a federal district court to hear claims over which it would not ordinarily have jurisdiction (e.g., state law claims that do not involve diversity jurisdiction) if those claims arise out of the same transaction or occurrence as a claim over which the court would have jurisdiction [28 U.S.C. Sec. 1367]. Federal district courts have original jurisdiction over all civil actions "arising under the Constitution, law, and treaties of the United States." [28 U.S.C. Sec. 1331]. Consequently, the court would have jurisdiction over the constitutional rights violation, and then would have supplemental jurisdiction over the battery since it was part of the same occurrence.

A reader of a quarterly magazine by the city government submitted a freelance article to the magazine critical of the mayor's dealings with the local business community. The magazine's editor thanked the reader for the article, but informed her that the magazine would not publish it because it was not the magazine's practice to publish articles critical of the mayor. The reader then filed a lawsuit against the city government on the basis of its failure to publish the article. Would the city government likely succeed in defending the lawsuit? A. Yes, because the city government's actions were in the context of its role as a publisher of the magazine. B. Yes, because the city government made a valid time, place, and manner restriction. C. No, because the city government made a content-based restriction. D. No, because the city government violated the Fourteenth Amendment Privileges and Immunities Clause.

A. Yes, because the city government's actions were in the context of its role as a publisher of a magazine. Discussion of correct answer:This answer choice is correct because when the government is the speaker, it may discriminate based on the content of the speech.

A student was injured was injured when he was hit by flying glass when the car a man was driving collided with the car a woman was driving at an intersection. The student sued the man in federal court for negligence and claimed that more than $75,000 was in controversy. The student and the woman were citizens of State A. The man was a citizen of State B. The man filed a motion to dismiss the complaint for failure to join an indispensable party and argued that the woman was an indispensable party because the woman, rather than the man, caused the accident. The court denied the motion. Was the court correct to deny the motion to dismiss the complaint for failure to join an indispensable party? A. Yes, because the woman was not a necessary party (a person to be joined if feasible). B. Yes, because the court have ordered the joinder of the woman without depriving the court of subject matter jurisdiction. C. No, because the student's claims against the man and the woman arose from the same event and shared common questions of law or fact. D. No, because joinder of the woman would have destroyed the complete diversity.

A. Yes, because the woman was not a necessary party (a person to be joined if feasible). Discussion of correct answer:Before a court can dismiss a case for failure to join an indispensable party, the court must find that joinder is required under Rule 19(a) and that joinder is not possible, for jurisdictional reasons. Here, the woman was not a necessary party. Complete relief could have been be awarded to the student in the action, and the man was not subject to multiple or inconsistent liability. The man was essentially arguing that the student should have sued the third party instead or in addition and that the woman was liable, not the man. Those circumstances do not make the woman a necessary party under Rule 19(a). Therefore, this answer is correct.

A manufacturer entered into a written agreement with a buyer, whereby the manufacturer agreed to build a high-performance racing boat for the price of $200,000. The boat was to be built according to specifications set forth by the buyer, and these specifications were to be incorporated into their agreement. The buyer was to pay for the boat upon delivery if it was in an operable condition. Before singing their written agreement, the manufacturer and the buyer orally agreed that the written contract would be of no effect until the buyer delivered to the manufacturer the plans and specifications and these plans had been assessed and approved by an engineer who worked with the manufacturer. When the buyer delivered the plans and specifications, the engineer refused to approve the plans because he believed that due tot he poor design of the hull, the boat would be very powerful but highly unstable. The manufacture then indicated to the buyer that the contract was no longer valid, and the buyer then brought an action against the manufacturer for breach of contract. In court, the mnafuahaturer's attorney sought to introduce evidence of the parties' oral agreement that the plans were to be approved by an engineer. The buyer's attorney objected on the grounds that the parol evidence rule precluded any evidence related to the oral agreement between the manufacturer and the buyer. Should the court admit the evidence? A. Yes, because the written agreement was to be effective only if and when the engineer approved the plans. B. Yes, because the engineer was not a party to the agreement between the manufacture and the buyer. C. No, because the parol evidence rule excludes the admission of evidence of a contemporaneous or prior oral agreement between parties that supports and integrated written agreement. D. No, because the absolute contractual obligations of the agreement were conditions upon the engineer's approval that was contradicted by the oral agreement.

A. Yes, because the written agreement was to be effective only if and when the engineer approved the plans. Discussion of correct answer:The parol evidence rule precludes the admission of evidence of a contemporaneous or prior oral agreement if it varies or contradicts the terms of an integrated written agreement. However, in this situation, the oral agreement between the manufacturer and the buyer regarding the need for the engineer's approval of the engineering plans prior to construction of the boat was an entirely separate agreement that did not alter the written agreement or contradict its terms. The parol evidence rule would not exclude evidence showing that a written agreement is not effective due to a failure to fulfill a condition precedent. Therefore, the manufacturer's evidence should be admitted.

Which of the following is admissible under an exception the hearsay rule? A. Recorded observations of a police officer in the criminal prosecution of a defendant. B. A report by the Federal Aviation Administration of the number of airplane accidents investigated over a given period of time. C. Factual findings made pursuant to authority granted by law that were complied by the defendant's wife. D. The official Department of Transportation's pamphlet of nonalcoholic drink recipes.

B. A report by the Federal Aviation Administration of the number of airplane accidents investigated over a given period of time. Discussion of correct answer:The report will be admissible as a public record. Public records are records, reports, statements, or data compilations, in any form, of public offices or agencies, setting forth: (1) the activities of the office or agency; (2) matters observed pursuant to duty imposed by law as to which matters there was a duty to report, excluding, however, in criminal cases, matters observed by police officers and other law enforcement personnel; or (3) in civil actions and proceedings and against the government in criminal cases, any factual findings resulting from an investigation made pursuant to authority granted by law, unless the sources of information or other circumstances indicate lack of trustworthiness. Here, a report of the number of airplane accidents investigated by the FAA would be a public record, because it sets forth the activities of the agency.

A man owned a 10-acre parcel of land with a farmhouse and a barn, which he converted to a 3-car garage. The man entered into a 3 year written lease with a tenant. The lease agreement provided that the tenant would be entitled to possession of the farmhouse, barn, and 10-acre parcel. The lease also contained an anti-assignment clause and a provision prohibiting subletting without the landlord's written consent. Which of the following arrangements would NOT violate the agreement? A. The tenant allowed his cousin to park his antique car in the garage for the sum of $50 per month. B. During the summer months, the tenant permitted his friends to swim and operate motor boats on a lake that was situated on the land leased from the man. C. The tenant allows a neighbor to come onto the property and remove large amounts of topsoil, which he uses in his landscaping business. D. The tenant allows his cousin to stay at the house while on college break in exchange for $25 per week.

B. During the summer months, the tenant permitted his friends to swim and operate motor boats on a lake that was situated on the land leased from the man. Discussion of correct answer: The issue of assigning and subleasing is often tested on the MBE. By definition, an assignment occurs if the tenant transfers the entire estate, or the balance thereof, to the third party. If a lesser estate is conveyed so that the tenant retains an interest in the leasehold, the transfer is properly classified a sublease. In this answer choice, the tenant's friends had a license (or mere permission) to use the lake for recreational purposes; there was no assignment or sublease. Because a license is not prohibited by the provisions in the lease, this is the correct answer choice.

A farmer died owning 10 acres of farmland, part of a once-large tract of farmland that was now almost completely surrounded by housing developments. In the last few years before his death, the farmer used the acres fronting a local road for his home and to run a small farmers' market, at which he sold produce grown on the "back" acres of his property. The farmer had no other access to these back acres other than via dirt road that ran from the back acres past his house and out to the public road. The farmer left his property to his daughter. The daughter sold the front four acres of the land, including the old homestead, to an entrepreneur and the remaining six acres to a construction company for the purposes of building townhouses. Neither deed contained any mention of an easement involving the dirt road. The entrepreneur planned to run a small home-based business on the property, supplying baked goods for her friend's catering business. The construction company's land had access to the public road only by means of the dirt road across the entrepreneur's land. The construction company had no problem moving construction equipment over the dirt road, but now sought to pave the road in order to begin bringing in customers to look at their model homes. The entrepreneur refused to allow the construction company to pave the road because she federated that the noise and traffic would cause her cakes to fall. If the construction company requests court permission to pave the road, in whose favor should the court rule? A. For the construction company, because it has an implied easement and the right to repair and maintain it. B. For the construction company, because its pavement of the road is within the reasonable contemplation of the parties. C. For the entrepreneur, because the deed to the construction company did not expressly grant an easement across the entrepreneurs property. D. For the entrepreneur, because the owner of the servant estate has the ultimate right to control the maintenance of an easement.

B. For the construction company, because its pavement of the road is within the reasonable contemplation of the parties. Discussion of correct answer:The owner of an implied easement has a limited right to upgrade the easement, and generally may not develop or upgrade it beyond the reasonable contemplation of the parties at the time the property was divided. This answer is correct because the daughter sold the land to the construction company knowing that it would be used for townhouses and that the homeowners would need access to the public road. It was thus reasonably within the contemplation of the daughter and the construction company that the road would eventually need to be paved.

A man was driving to work in State A when he was hit from behind by a car being driven by a tourist. The man is a State A citizen and the tourist is a State B citizen. The man sued the tourist in the federal court in State B where he claimed $30,000 in physical damages and $100,000 in mental anguish. Under State A and federal law, damages for mental anguish were proper. However, a recent State B Supreme Court decision stated damages for mental anguish were not available for car accidents. The tourist filed a motion to dismiss, claiming the State B federal court did not have jurisdiction over the claim. How should the court rule? A. Grant the motion, because the accident occurred in State A. B. Grant the motion, because of the recent State B decision. C. Deny the motion, because the tourist is a citizen of State B. D. Deny the motion, because the amount in controversy is over $75,000.

B. Grant the motion, because of the recent State B decision. Discussion of correct answer:The Erie doctrine states that a federal court that hears a state law claim in a case based on diversity or supplemental jurisdiction must apply the substantive law of the state in which the court sits [Erie R.R. Co. v. Tompkins, 304 U.S. 64 (1938)]. Here, the federal court sits in State B, so it would apply State B law. Under the recent State B decision, damages for mental anguish are not available for car accidents. This means that the $100,000 claim for mental anguish cannot be raised, leaving the amount in controversy only $30,000. Consequently, the federal court would not have subject-matter jurisdiction over the controversy and should dismiss the claim.

A high school English teacher was the author of several books on teaching English to the disadvantaged. One day, a reporter for the local newspaper came to the high school to do a story on the school's basic competency scores, which were well below the state average. The reporter entered the teacher's classroom while she was absent. The reporter glanced at the teacher's laptop and noticed an email message addressed to a local merchant voicing her dissatisfaction with a product she had purchased. The email contained numerous punctuation and spelling mistakes. The reporter hurriedly printed out the email message and departed. The reporter called the teacher prior to the publication of his feature story that was to include the teachers email verbatim. The teacher explained to him that the email was genuine, but that she was using an error laden version as an exercise for her English class. The reporter responded, "That's a likely story, lady", and hung up. The teacher was extremely embarrassed when the story was published in the newspaper and endured a good deal of ribbing and even ungracious innuendo from her colleagues, and she filed a defamation action against the newspaper. What will the teacher have to prove in order to prevail in her defamation suit against the newspaper? A. Malicious publication, because the school teacher is a public figure. B. Negligent publication, because the school teacher is a private person and this is a matter of public concern. C. Reckless publication, because the school teacher has authored several books. D. Publication only, because the school teacher is a private person and this is a matter of private concern.

B. Negligent publication, because the school teacher is a private person and this is a matter of public concern. Discussion of correct answer:To prevail in an action for defamation, the teacher will be required to establish that the newspaper article: 1) was a false, defamatory statement of fact; 2) was reasonably understood as relating to the teacher; 3) was intentionally or negligently published to a third party; 4) caused her damage; and 5) was made with the requisite degree of fault as to the truth or falsity of the statement. Generally, if an allegedly defamatory statement involves a matter of public concern (such as the performance of public school teachers in this case), a private person suing a media defendant for defamation must show that the defendant was at least negligent with regard to the truth or falsity of the printed statement. Note that although the teacher was the author of several books on teaching English to the disadvantaged, she will not be considered a public figure, as she has not achieved an all-pervasive fame, attempted to use the media to gain notoriety, nor injected herself into the public arena to attempt to direct public policy. Therefore, to recover damages for defamation, the teacher must establish that the newspaper and/or its agent, the reporter, were negligent in investigating the truth or falsity of the statement. As such, this answer is correct.

An investor is currently on trial for sexual assault of an elderly woman. The investor's girlfriend is called to the stand by the prosecution for the purpose of incriminating the investor. During cross-examination, the investor's attorney asks the girlfriend, "Isn't it true that you have also been charged with being an accessory to the sexual assault and your boyfriend is currently being tried for?" The girlfriend answers in the affirmative. The investor's attorney then asks, "And isn't it true that the prosecutor agreed to drop those charges in exchange for testimony that would incriminate your boyfriend?" The prosecution objects. Should the objection be sustained? A. No, because the girlfriend's answer would constitute an opposing party's statement. B. No, because the girlfriend's answer would constitute proper impeachment evidence. C. Yes, because in order to impeach a witness in this manner, there must be proof of a conviction. D. Yes, because it is against public policy to admit evidence that is essentially an offer to compromise.

B. No, because the girlfriend's answer would constitute proper impeachment evidence. Discussion of correct answer:The bias of a witness is relevant to his or her truth and veracity, because a witness's testimony may be influenced by a personal relationship with a party or a personal interest in the outcome of the case. Therefore, inquiring on cross-examination as to whether a witness is biased is always a permissible form of impeachment. Here, the cross-examination of the girlfriend establishes that the girlfriend has a motive for offering testimony that is favorable to the prosecution and that she therefore has a motive to lie. Thus, the girlfriend may be impeached to show her bias and lack of credibility and the objection should be overruled.

A woman loved to watch her neighborhood football games. According to the neighborhood tradition, whenever a player scored a touchdown, the player would immediately rush full-speed at one of the fans on the sidelines and jokingly tackle him or her. One Saturday, as the woman and her neighbors are enjoying a game, a player scores a touchdown. He immediately looks for his little brother Ron the sidelines, intending to tackle him. The woman, standing directly behind the player's brother, sees the player looking in their direction. Exhilarated by the play, she jokingly calls out, "Come on, big boy!" The player charges at her. Realizing that she has become the player's target, the woman ducks, but the player succeeds in tackling her. The woman falls to the ground and sustains a concussion. If the woman now brings suit against the player for her injuries, will she recover? A. No, because injuries sustained during sporting events are foreseeable. B. No, because the woman consented to the player's contact. C. Yes, because the woman did not expressly consent to the contact. D. Yes, because the woman suffered a harmful and offensive contact.

B. No, because the woman consented to the player's contact. Discussion of correct answer:A defendant will not be liable in a battery action if the plaintiff consented to the contact in question. Consent is defined as a plaintiff's willingness in fact that the tortious contact take place. Consent may be granted expressly, in words, or it may be implied from the plaintiff's conduct. Here, as a regular watcher of neighborhood football games, the woman was clearly aware of the neighborhood tradition whereby players, after making a touchdown, would tackle fans on the sidelines. As such, it is arguable that merely by attending the game, the woman impliedly consented to such contact. However, even if the woman is not found to have impliedly consented to being tackled, the facts indicate that she overtly called out to the player, "Come on, big boy!" By her words, the woman expressly consented to being tackled. The woman's battery claim will fail. Therefore, answer choice (C) is incorrect. Choice (A) is incorrect because this is not a negligence situation, rather an intentional act. Choice (D) is incorrect because although a harmful and offensive contact may have occurred, the woman's presence at the game and actions show her consent to the physical touching.

Over 40 years ago, a man obtained title to a parcel of land through adverse possession. The record owner of the land has long since passed away and was not survived by any family members. For the past 40 years, the man has used the land as his primary residence. However, because the man is now elderly, he wishes to sell the land to a buyer road move into an assisted-living facility. Under the majority rule, can the man convey marketable title in the land to the buyer. Under the majority rule, can the man convey marketable title in the land to the buyer? A. No, because the man has held the land by adverse possession for a period of less than 50 years. B. No, because title acquired through adverse possession is not marketable absent a court judgment. C. Yes, because the acquisition of title by adverse possession generally does not destroy marketability of title. D. Yes, because the risk that the record owner will sue for title to the land is remote.

B. No, because title acquired through adverse possession is not marketable absent a court judgment. Discussion of correct answer:All contracts for the sale of real property include an implied promise to convey marketable title. Marketable title is title that is reasonably free from doubt in both fact and law. Title is not reasonably free from doubt if it contains any defects in the chain of title, such as adverse possession. Traditionally, title acquired by adverse possession is unmarketable until the adverse possessor obtains a judgment in his favor in a quiet title action. Without a quiet title judgment in his favor, the man's title remains unmarketable.

The US Congress enacted the Women and Children's Nutrition Act and appropriated funds for disbursement to the states for programs for women with children under the age of 6. The purpose of the act was to provide nutrition and basic health care to mothers and children at risk because of poverty and substance abuse. Funds were disbursed directly to state departments of income assistance for use in social welfare programs specifically set up to provide nutritional support, including counseling and foodstuffs, and health care in the form of vaccinations and medical exams. As a condition of receipt of the funds, states were required to document substance abuse counseling in the health care component of the program. Counseling on the dangers of drug or alcohol abuse was mandatory, even if clients did not have drug or alcohol problem. Which of the following is the most accurate statement about the Act? A. The Act constitutionally regulates for the general welfare under the spending power. B. The Act constitutionally conditions receipt of federal funds on mandatory substance abuse counseling. C. The Act unconstitutionally infringes upon clients' right to privacy guaranteed under the Fourteenth Amendment. D. The Act unconstitutionally requires states to provide counseling that is not reasonably related to the purposes of the act.

B. The Act constitutionally conditions receipt of federal funds on mandatory substance abuse counseling. Discussion of correct answer:By the exercise of its spending power [U.S. Const. Art. I, Sec. 8, cl. 1], the U.S. Congress can condition receipt of federal funds on the states' voluntary conformance to federal regulations. The condition requiring mandatory counseling of clients about the dangers of substance abuse is reasonably related to the goal of the spending program, i.e., to protect and preserve the health of women and young children.

A farmer and his business associate owned a large farm as joint tenants with right of survivorship. Their primary crop was corn. Following a general decline in the market price of corn, the business associate suggested making wheat their primary crop. The farmer agreed that such a change Ould make sense under the circumstances, and he and the business associate implemented the change. However, shortly before the first wheat crop was to be harvested, the farmer told his business associate that he preferred growing corn, and that he would be conveying his 1/2 interest in the farm to an interested buyer. The farmer's deed of conveyance to the buyer made no mention of the wheat crop growing on the land. What inters will the buyer own in the farm and/or the wheat crop following the farmer's conveyance of his 1/2 joint tenancy interest? A. The buyer will own 1/2 share of the farm as a tenant in common with the business associate, and the buyer will not own a 1/2 interest in the wheat crop growing on the land. B. The buyer will own 1/2 share of the farm as a tenant in common with the business associate, and the buyer will own a 1/2 interest in the wheat crop growing on the land. C. The buyer will own 1/2 share of the farm as joint tenant with the business associate, and the buyer will own 1/2 interest in the wheat crop growing on the land. D. The buyer will own 1/2 share of the farm as a joint tenant with the business associate, and the buyer will not own a 1/2 interest in the wheat crop growing on the land.

B. The buyer will own a one-half share of the farm as a tenant-in-common with the business associate, and the buyer will own a one-half interest in the wheat crop growing on the land. Discussion of correct answer:A joint tenancy is severed if one joint tenant conveys his interest, whether voluntarily or involuntarily. If a joint tenancy is severed, the unities of time and title are destroyed, thereby creating a tenancy in common between the other tenant and the conveyee. In this case, upon the farmer's voluntary conveyance of his joint tenancy interest in the farm to the buyer, the joint tenancy was severed, and the business associate and the buyer became tenants in common, each owning a one-half interest in the farm. Annual growing crops produced by human cultivation and labor are known as fructus industriales, or emblements. Under the common law, fructus industriales are considered to be part of the land, and will pass with a conveyance of the land unless the grantor expressly reserves the crop in the deed of conveyance. Thus, the wheat crop growing on the farm will be considered part of the farmer's conveyance to the buyer. As such, this answer is correct.

In preparing for her law school examinations, a student invited a few classmates to her apartment for a study group. The student, who loved spiders, kept a pet tarantula caged in her apartment. When the student purchased the tarantula, she had the poisonous venom removed so it was harmless. During the study group session, the student brought out the cage with the tarantula to show her friends. When she was placing the cage down on the kitchen counter, the student carelessly left the cage door slightly ajar. Moments later, the tarantula crawled out of the cage and crept toward one of the classmates. The classmate, who was deathly afraid of spiders, saw the tarantula and tried to run away. As she did so, she tripped over a chair and fell down, fracturing her wrist. If the classmate files suit against the student to recover damages for her injury, who will prevail? A. The classmate, because the student was negligent in leaving the cage door open. B. The classmate, because the student is strictly liable. C. The student, because the spider did not directly cause the classmate's injury. D. The student, because a spider is not a wild animal.

B. The classmate, because the student is strictly liable. Discussion of correct answer:In analyzing a Torts question involving animals, it is first necessary to determine whether the animal is domesticated or wild. A possessor of a wild animal is subject to strict liability for the harm that results even though the possessor has exercised the utmost care to confine the animal. The word "animal" is used in a broad sense to include birds, fish, reptiles, insects, and other living creatures. Thus, a tarantula would be considered a wild animal, and the student is strictly liable for the resulting harm. As such, this answer choice is correct.

The defendant is the owner of a company that is engaged in the manufacture and sale of a variety of pet supplies. The defendant's company manufactures such products as pet furniture, toys, beds, collars, leashes, cages, vitamins, etc. These items are distributed to pet stores throughout the US and Europe. For the past ten years, the company has conducted its operations from a large factory located on the outskirts of town. One of the defendant's biggest selling products is specially manufactured high-frequency dog-calling whistles. These whistles are sold to dog-training schools and kennel divisions of many police departments. Although these whistles are not audible to people, they are audible to dogs over considerable distances. Recently, the Plaintiff purchased an undeveloped lot near the defendant's factory. On her property, the plaintiff constructed a pet hotel, which was used as a boarding kennel for dogs and cats. This boarding facility was situated about 100 yards from the defendant's factory. Unbeknownst to the Plaintiff, high-frequency sound wavs rotten emitted from the defendant's factory when dog-calling whistles were being tested. These sound waves caused the plaintiff's dogs to howl uncontrollably for many hours during the day and seriously interfered with the operation of her business. The plaintiff now brings an action against the defendant and the defendant's company to recover damages for the interference with her business caused by the high-frequency sound that reaches her kennel, under a theory of private nuisance. Judgment for whom? A. The defendant, because the plaintiff came to the nuisance after his factory had already been in operation for a number of years. B. The defendant, because the plaintiff's business is abnormally sensitive to harm caused by the high-frequency sound waves. C. The plaintiff, because the high-frequency sound waves constitute a trespass to the plaintiff's premises. D. The plaintiff, because the high-frequency sound waves have seriously interfered with the operation of her business.

B. The defendant, because the plaintiff's business is abnormally sensitive to harm caused by the high-frequency sound waves. Discussion of correct answer:The defendant would not recover for nuisance if her business was abnormally sensitive to harm caused by high- frequency sound waves. A private nuisance is a thing or activity that substantially and unreasonably interferes with the plaintiff's use and enjoyment of her land. However, the plaintiff cannot, by devoting her land to an unusually sensitive use, complain of a nuisance based on conduct that would otherwise be relatively harmless. In this case, the defendant's conduct of testing dog whistles would otherwise be relatively harmless, inasmuch as it cannot be heard by the human ear. The plaintiff uses her land for an unusually sensitive use; therefore, she cannot recover for private nuisance in this case.

A business executive offered to convey his country estate to his son if he would work full time in the executives prepared a deed conveying the country estate to the son in fee simple absolute and showed it to him, together with a notarized statement signed by the executive containing the executive's promise to convey the country estate upon the son's completion of five years of service with the company. The son agreed to the terms. The executive intended to place the deed in his safe, but became distracted and left his study leaving the deed on his desk. The son's girlfriend soon came into the study and spotted the deed on the desk. After skimming and notarized statement, she hurriedly placed the deed in an envelope addressed to the son and mailed it out. The son received the deed the following dyad promptly recorded it. Several weeks later, the son sold the country estate to a doctor acing as the principal of a group looking for a property on which to establish a country club. The doctor recorded the deed immediately after the conveyance. Eventually, the son told the executive that he had decide. not to work int eh corporate world. The executive, upset, sold the country estate to a real estate developer, who recorded the deed immediately thereafter. When the developer visited the country estate shortly after the conveyance, he found heavy equipment in the process of carving out a golf course on the property. In a suit by the developer to quiet title tot eh country estate, in whose favor is the court most likely to rule? A. The developer, because a deed from the original grantor is superior to a deed from a grantee. B. The developer, because the executive kept the deed after discussing the country estate with the son. C. The doctor, because he recorded first. D. The doctor, because the executive is estopped from denying delivery to the son.

B. The developer, because the executive kept the deed after discussing the country estate with the son. Discussion of correct answer:When the grantor keeps the deed to the property, there is a presumption that no valid delivery was made. Here, the executive did not have a present intent to convey the country estate to the son; he intended to convey the property to the son only after the son had worked in the executive's company for a period of five years, and the executive did not deliver the deed to the son. Instead, the son acquired the deed essentially by theft, when his girlfriend removed it from the executive's desk and placed it in the son's possession (through mailing it). Parol evidence, including the executive's notarized statement regarding the conveyance of the country estate, is admissible to prove the grantor's present intent. As such, the court will likely find that the developer is the owner of the country estate.

Two neighbors owned adjoining lots in the central portion of a city. Each of their lots had an office building built upon it. one of the neighbor's decided to tear down the existing building on her lot and to erect a building of greater height. She obtained all governmental approvals required to pursue her project. After she had torn down the existing building, she proceeded to excavate deeper. She used shoring that met all local, state, and federal safety regulations, and the shoring as placed in accordance with those standards. Halfway through the construction of the new building, she received a letter from the adjoining landowner notifying her that cracks were developing in the building situated on his lot. The excavation landowner took the position that any subsidence suffered by Thea adjoining landowner was due to the weight of his own existing building, and correctly asserted that none would have occurred had the neighbor's soil been in its natural state. The landowner continued to excavate, and the building on the neighbor's lot suffered extensive damage, requiring the expenditure of $750,000 to remedy the defects. Which of the following is the most accurate statement concerning the aggrieved neighbor's action to recover er damages from the excavating landowner? A. The excavating landowner is liable, because she removed necessary support for her neighbors lot. B. The excavating landowner cannot be held strictly liable based solely on the fact that support was removed, but may be held liable if negligence is proven. C. Once land is improved with a building, the owner cannot invoke the common law right of lateral support. D. The excavating landowner's only obligation was to satisfy all local, state, and federal safety regulations.

B. The excavating landowner cannot be held strictly liable based solely on the fact that support was removed, but may be held liable if negligence is proven. Discussion of correct answer:The right of a landowner to have her land supported laterally by the neighboring land is an absolute right inherent in the land itself. The right of lateral support extends only to the land in its natural condition; the right of lateral support does not include the right to have the additional weight of artificial structures supported. Thus, in this case, the excavating landowner will be strictly liable for damages to buildings on the neighbor's adjacent land caused by excavations only if it can be proven that the land would have suffered damage in its natural state. Otherwise, the excavating landowner will be liable for damages only if the excavation was done negligently. Therefore, this answer is correct.

A tenant rented an apartment and entered into a written lease agreement for a term of 6 years with a monthly rental fee of $1,500. the tenant resided in the apartment for 3 years. Then, the tenant drafted a written agreement in which he transferred his entire interest for 2 years to a friend. According to the written instrument, the friend was obligated to directly pay the tenant $2,000 per month for the term of occupancy. For the next 6 months, the friend paid the tenant $2,000 each month, of which the tenant paid the landlord $1,500. During the seventh and eighth months, the friend continued to make his $2,000 payments to the tenant. However, the tenant did not make any rental payments to the landlord for those 2 months. After not receiving his rental payments, the landlord went to the apartment and found the friend in possession. The landlord then sued the friend for the 2 months' rent that had not been paid. For whom should the court enter a judgment? A. The friend, because he made his monthly rental payments to the tenant. B. The friend, because as a subtenant he is not obligated to the landlord. C. The landlord, because the tenant transferred the entire interest. D. The landlord, because there is privity of contract between landlord and the friend.

B. The friend, because as a subtenant he is not obligated to the landlord. Discussion of correct answer:A subtenancy is created when a tenant transfers his right of possession for a time shorter than the remainder of the leasehold, so that he retains a reversion within the leasehold. Therefore, the subtenant is neither in privity of estate nor privity of contract with the landlord. Lacking privity, a subtenant is not liable to the landlord to pay rent. In this case, the tenant had three years remaining in the lease term and transferred only a two-year interest to his friend. Thus, the tenant subleased the premises, and the subtenant (the friend) bears no obligation to the landlord. Therefore, this answer choice is correct.

A married couple entered into a two year lease. The neighborhood consisted primarily of vehicle repair shops and older couples. For the first year, the landlord never visited the property and the couple paid the rent as directed. In the second year, the couple had a child and decided to use the spare room beside the master bedroom as the nursery. Rather than leave the mater bedroom each time to enter the nursery, the couple decided to hire a local man to add a doorway in the wall between the two rooms. The man, a certified building contractor, checked the wall to make sure it was not a "load bearing wall", to which local ordinances preclude any kind of alterations. When the lease was about to end, the landlord inspected the premises and noticed the doorway between the two rooms. He remarked that it was a nice touch, and even conceded that it made the home more valuable, because the neighborhood, particularly over the pat year, had begun to accommodate many younger couples starting families. The couple vacated the property at the expiration of the lease without restoring the wall. The landlord then sent the couple a bill for the estimated cost to remove the doorway in the wall. The couple refused to pay the bill, and the landlord filed suit to recover this amount.. Assuming the common law applies, what is the likely outcome of the case? A. The landlord will prevail, because adding a doorway violates the lease. B. The landlord will prevail, because the addition of the doorway is waste. C. The couple will prevail, because the addition of the doorway increased the property value. D. The couple will prevail, because they were long-term tenants.

B. The landlord will prevail, because the addition of the doorway is waste. Discussion of correct answer:A tenant is liable for committing waste whether the waste diminishes or increases the value of the property. Ameliorative waste is waste which increases the value of the property. Pursuant to the common law, ameliorative waste is still actionable by the landlord. Here, the couple has committed (ameliorative) waste, and they will be liable to the landlord for damages. Note that under the modern rule, a life tenant is now allowed to commit ameliorative waste if the market value of the remainderman's interest is not impaired, and either: a) it is permitted by the remainderman; or b) a substantial and permanent change in the neighborhood has deprived the property of a reasonable current value. However, since this questions specifies that the common law applies, this is the correct answer choice.

Late one evening, a teenager boy and his 15 year old girlfriend were leaving a public library. As they walked through the dimly lit parking lot to get their bikes, an adult man, who was visibly intoxicated, emerged from behind a parked car and ran up to them. The man knocked the boyfriend on the ground and hit him over the head with a wrench, causing the boyfriend to lose consciousness. The man then forced the girlfriend into his car. Moments later, the boyfriend regained consciousness, and the man fled the scene. The man was subsequently charged with assault with the intent to commit rape. Which of the following would provide the man with his best defense? A. The man thought that the girlfriend had consented. B. The man did not intend to rape the girlfriend. C. The man's intoxication at the time negated the required general intent. D. It is impossible to prove that the man was the perpetrator, because the parking lot was dimly lit.

B. The man did not intend to rape the girlfriend. Discussion of correct answer:Assault with the intent to commit rape is a specific intent crime. If the man establishes that he did not actually intend to rape the girlfriend, this evidence would provide him with the best defense.

A buyer and seller entered into a written agreement for the sale of 100 widgets at $1.50 each. The contract provided that the widgets would be shipped in ten cartons, each containing ten widgets. Because the buyer knew that the cartons of widgets would be moved several times before they were used, it was important to him that the widgets be individual wrapped. For this reason, the buyer requested that each widget be individually packed and shipped in its own carton. The seller agreed to this modification over the phone. Shortly thereafter, the seller shipped the widgets to the buyer in ten cartons, as originally agreed. The buyer rejected the shipment and demanded new widgets in individual cartons. The seller refused. Is the seller in breach of contract? A. Yes, because contracts can always be modified. B. Yes, because the original contract can be orally modified without additional consideration. C. No, because the modification must be in wiring when the sale of goods is involved. D. No, because there was no consideration for the seller's agreement to change the mode of shipping.

B. Yes, because the original contract can be orally modified without additional consideration. Discussion of correct answer:Section 2-209 of the Uniform Commercial Code (UCC) provides that an agreement for the sale of goods may be modified without additional consideration. Therefore, this is the correct answer choice.

A once quant town was ill equipped to absorb the number of new residents who flocked to the area in the early 2000s. Overwhelmed by the new issues facing the town, the tiny town council acted quickly to expand the town government. Pursuant to a new ordinance, one of the requirements for those placing their names on the ballot is the execution of a loyalty oath. A resident of the town plans to run for office and fulfills all of the other requirements in a timely manner, but declines to sign the loyalty oath. If the town clerk declines to play the candidate's name on the ballot and the candidate sues in federal court for the right to stand for election, how is the court likely to rule regarding the ordinance requiring candidates to sign a loyalty oath? A. The ordinance is unconstitutional because it denies the candidate access to the ballot. B. The ordinance is constitutional if it requires only that the candidate support the Constitution of the US. C. the ordinance is constitutional if it requires only that the candidate swear that she has not been and never will be a member of any subversive organization committed other overthrow of the government by force or violence. D. The ordinances is constitutional if it requires only that the candidate swear that she has not and will not criticize any government institutions.

B. The ordinance is constitutional if it requires only that the candidate support the Constitution of the United States. Discussion of correct answer:Loyalty oaths required as a condition of public employment or public service in an elected office may not be vague or overbroad. It is constitutionally permissible for oaths to require the affiant to promise to support the Constitution. Thus, a loyalty oath requiring that the affiant pledges to support the Constitution of the United States is permissible, as it is essentially an oath to uphold the constitutional process.

The day before Christmas, a shuttle-bus driver began making trips to and from the airport beginning at 4:00 am, and by 5:00 pm he had already worked 13 hours. As tired as he was, the shuttle bus driver continued picking up and dropping off passengers, heading out with a nearly full shuttle to retrieve his last group of passengers and return to the airport. As the shuttle pulled up to the next bus stop, a masked man shoved his way past the waiting passengers and onto the shuttle wielding a large handgun. The masked man loudly demanded that the driver and passengers empty their wallets and hand over their cash. At that very moment, a passenger who had been waiting to board the shuttle dove on top of the gunman, knocking him to the ground. As the passenger attempted to wrestle the handgun from the masked man, a shot rang out. The gunman jumped to his feet and scrambled out the door. The passenger lay on the floor of the shuttle groaning, having been shot in the stomach. If the passenger sues the shuttle-bus driver, what is the shuttle-bus driver's best defense? A. The shuttle-bus driver owed passengers a duty only to use reasonable care to prevent injuries caused by his employees. B. The shuttle-bus driver owed passengers a duty only to protect them from foreseeable danger. C. The shuttle-bus driver had no duty to control the acts of third parties. D. The shuttle-bus driver was not the legal cause of the passenger's injury.

B. The shuttle-bus driver owed passengers a duty only to protect them from foreseeable danger. Discussion of correct answer:Generally, an individual does not have a duty to protect persons from third-party criminal conduct. However, where the persons are business invitees, a special relationship may be found to exist, thereby triggering a duty to protect. Depending upon the jurisdiction, the burden to protect hinges on the degree of foreseeability of the third-party criminal conduct. Some jurisdictions impose a duty to protect only where there have been prior similar incidents, making the third-party criminal conduct especially foreseeable. Here, the shuttle-bus passengers were business invitees of the shuttle-bus driver, and the shuttle-bus driver certainly owed them a duty of reasonable care with regard to any of his own acts that might pose an unreasonable risk of harm. The driver also owed passengers a duty of care with regard to acts of passengers that posed foreseeable risks to other passengers and/or third parties and that the shuttle-bus driver could reasonably control. However, the shuttle-bus driver most likely did not have a duty to protect passengers from such unforeseeable harm as an armed robbery by a stranger boarding the bus. Therefore, the shuttle-bus driver's best defense is that he owed passengers a duty only to protect them from foreseeable danger.

A chef had equipped his home kitchen with the latest cutting-edge appliances, including a brand-new cappuccino machine. One night when the chef was at work and the nanny was at home with the chef's two small children, the nanny decided to use the cappuccino machine. The nanny knew that the chef preferred that she not tamper with his equipment other than to use the microwave oven to reheat the dinners which he prepared every day for his children. However, it was a particularly chilly night, and the many was relatively certain that she knew how to operate the machine. The nanny was unaware, however, that the machine had been recalled by the manufacturer for the design defect that caused one of the handles to overheat, creating the risk of severe burns. The chef knew of the recall but had not thought to warn the nanny. While using the cappuccino machine, the nanny burned her hand on the overheated handle and had to seek medical treatment. If the nanny files a strict products liability action against the manufacture of the cappuccino machine, is she likely to prevail? A. Yes, if the nanny demonstrates that the defect was the result fo the unreasonable design by the manufacturer. B. Yes, because the cappuccino machine was defective when it left the manufacturer. C. No, because the nanny was not supposed to use the chef's equipment. D. No, because the nanny was not the purchaser of the cappuccino machine.

B. Yes, because the cappuccino machine was defective when it left the manufacturer. Discussion of correct answer:Commercial suppliers at all levels of the distribution chain are subject to product liability claims for defective products that cause injury to proper plaintiffs. In this case, the defect in the cappuccino machine is clear. A commercial manufacturer of a defective product, whether it be a manufacturing defect or a design defect, is liable to proper plaintiffs. Thus, the pertinent issue is whether the nanny is a proper plaintiff or whether the manufacturer has any valid defense. A proper plaintiff in a strict products liability case is the purchaser or consumer of the product, as well as his family, friends, guests, and employees. The nanny would almost certainly fall within one of these categories, and is therefore a proper plaintiff. Defenses to strict products liability include misuse that is not foreseeable and assumption of the risk. Similarly, if a third party's tortious conduct is an unforeseeable intervening force that supersedes the defendant's wrongdoing, then the defendant will not be liable. Here, however, the nanny did not misuse the machine, and she was not aware of the defect and did not willingly assume the risk. Although the chef did not want the nanny to use his equipment, her use of the cappuccino machine was not unforeseeable or tortious. Therefore, the nanny is likely to prevail in her strict products liability claim against the manufacturer, making this the correct answer choice.

A computer programmer agreed to write a custom accounting program for an accountant for a "reasonable fee." The written agreement provided that the accountant had the right to terminate the contract at any time before the programmer finished the program "if a suitable substitute computer program became available on the market." After the programmer had logged about 50 hours working on the program, the accountant gave notice of cancellation. No substitute program had come onto the market, but the accountant had decided it would be cheaper to continue his current methodology. May the programmer enforce his right to payment under the contract? A. Yes, because the programmer can use under the doctrine of promissory estoppel. B. Yes, because the contract, which is not illusory, is enforceable. C. No, because the accountant's right to cancel made the contract illusory. D. No, because the reasonable-fee provision rendered the contract too uncertain to enforce.

B. Yes, because the contract, which is not illusory, is enforceable. Discussion of correct answer:A promise to perform that leaves performance to the discretion of the promising party is an illusory promise and does not constitute consideration. Here, the accountant's right to cancel was not left within the accountant's own discretion; rather, the accountant would have the right to cancel only if a suitable program became available on the open market. Therefore, the contract was not illusory and is fully enforceable in all other respects.

A man sued a contractor for negligently igniting a gas line in the man's house and causing a fire. The man sued the contractor under diversity jurisdiction in the US District Court for the District of State A. The man filed the complaint on February 1. Under the applicable state statute of limitations, the man had to commence an action against the contractor before the expiration of 2 years after the cause of action accrued. The man's cause of action accrued on February 3, two years before the year in which the man sued the contractor. The man had the contractor served on February 4, three days after the complaint was filed. Under the laws of State A, a civil action was "commenced" by the service of the complaint on the contractor. The contractor filed an answer in which the contractor raised the statute of limitations as a defense and then filed a motion for summary judgment based upon that defense. The court granted the motion. Was the court correct to grant the motion for summary judgment based upon the statute of limitations defense? A. Yes, because the contractor preserved the defense by including it int he contractor's answer. B. Yes, because the state's method of determining when an action is "commenced" controlled in a federal diversity case. C. No, because the federal civil action was "commenced" upon the filing of the complaint. D. No, because federal courts sitting in diversity apply federal procedures.

B. Yes, because the state's method of determining when an action is "commenced" controlled in a federal diversity case. Discussion of correct answer:The Supreme Court of the United States has addressed this issue twice, first in Ragan v. Merchants Transfer and Warehouse Co. [337 U.S. 530 (1949)] and again in Walker v. Armco Steel Corp. [446 U.S. 740 (1980)]. The general rule is that a federal court sitting in diversity will apply a controlling federal procedural statute or rule, even if the state rule or law is different. Here, however, Federal Rule of Civil Procedure 3's definition of "commencement" of the case as of the filing controls other time periods within the rules but was not intended to affect state statutes of limitation. The state's rule on when an action is "commenced" for limitations purposes thus controlled, and the motion for summary judgment was properly granted. Therefore, this answer is correct.

Among the beliefs of a particular religious cult is that ritual, voluntary suicide is to be experienced and encouraged. The State of Q prosecuted the leader of the cult under a state statute that made it a crime for anyone to exhort a minor to commit suicide under the circumstances in which a minor is likely to do so. The cult leader had admittedly exhorted minors to commit suicide under such circumstances. Most recently, however, the cult leader has pledged not to exhort minors in this manner. If the cult leader defends against the prosecution solely on the ground that the state statute interferes with his freedom of religion, can be constitutionally be convicted? A. Yes, because the state may show that its statute is necessary to accomplish the compelling state interest of preventing suicides. B. Yes, because the statute is neutral and generally applicable to everyone rather than being enacted to discriminate against a particular religion. C. No, because purely religious actives pursued in good faith are immune from governmental regulation. D. No, because only the person attempting suicide has committed an overt act that the state may punish.

B. Yes, because the statute is neutral and generally applicable to everyone, rather than being enacted to discriminate against a particular religion. Discussion of correct answer:The Supreme Court has held that, when a statute is neutral and generally applicable, it does not violate the Free Exercise Clause even if it prohibits religious conduct [Employment Division v. Smith, 494 U.S. 872 (1990)]. That is the case here, where there is no evidence that the state had any intent to interfere with the religious beliefs in question. Thus, this is the best choice because it correctly explains why the cult leader's Free Exercise Clause claim will fail.

Congress passed a bill that amended the Pell Grant law, creating five year residency requirement in order to qualify for an award of a Pell Grant. During the debate on the bill, the main sponsor of the bill stated that the purpose of the bill was to ensure the financial viability of the Pell Grant plan. This purpose was also made explicit in the preamble of the bill. The President of the US signed the bill into law. A group of alien students who were denied aid under the Pell Grant program sued to challenge the new law. Assuming proper standing, how should the court rule? A. In favor of the alien students, because under the rational basis test, the five year residency requirement was not rationally related to any legitimate interest. B. In favor of the alien students, because as alienage is a suspect class, under strict scrutiny the five year residency requirement was not the least restrictive means available to further a compelling interest. C. Against the alien students, because the federal classifications that make distinctions between clients are not subject to strict scrutiny test, as the US congress has broad plenary power to regulate immigration. D. Against the alien students, because under intermediate scrutiny the five year residency requirement was not substantially related to an important interest.

C. Against the alien students, because federal classifications that make distinctions between aliens are not subject to the strict scrutiny test, as the U.S. Congress has broad plenary power to regulate immigration. Discussion of correct answer:Alienage is a suspect class; thus, ordinarily, strict scrutiny analysis is used. However, federal classifications based upon alienage are not subject to the strict scrutiny test because the U.S. Congress has broad plenary power to regulate immigration. In Mathews v. Diaz [426 U.S. 67 (1976)], the U.S. Supreme Court upheld a federal law that established a five-year residency requirement for federal Medicare benefits that disqualified many resident aliens. As benefits under the Pell Grant program can be equated to Medicare benefits, this answer is correct

A male and female teenager wanted to go to movie, but neither had enough money. They went into a local coffee shop and saw that there was a tip jar on the counter. The female teenager said, "I bet there's enough money in the tip jar for us to go to a movie and get some pizza afterwards." The male teenager agreed and suggested that the female teenager create a diversion so that he could take the tip jar. The female teenager agreed and said she would meet the male teenager a few blocks away. The female teenager ordered a coffee and when it came, she purposefully knocked it over onto the barista. While the female teenager helped the barista clean up, the male teenager grabbed the tip jar and ran out of the coffee shop. As defined in this jurisdiction, the crime of conspiracy requires an overt act in furtherance of the conspiracy. If prosecuted, the female teenager should be found guilty of which of the following? A. Conspiracy, but not larceny. B. Larceny, but not conspiracy. C. Both larceny and conspiracy. D. Only larceny, because the conspiracy will merge into the completed crime.

C. Both larceny and conspiracy. Discussion of correct answer:A conspiracy is an unlawful criminal combination between two or more people who agree to commit an unlawful act. Larceny is the taking and carrying away of the tangible personal property of another with the intent to permanently deprive the owner thereof. In this case, the male and female teenager agreed to commit an unlawful act (the theft of the tip jar). In addition, both committed overt acts in furtherance of the conspiracy; the female teenager created a distraction, while the male stole the tip jar. Thus, the female teenager is clearly guilty of conspiracy. And, because a conspirator is guilty of the crimes committed by co-conspirators that are a foreseeable outgrowth of the conspiracy and are committed in furtherance of the conspiracy, the female teenager is also guilty of larceny, even though it was the male teenager who actually stole the tip jar. As such, the female teenager should be found guilty of both conspiracy and larceny.

A bar manager told one of his bartenders that there was a new type of home blender on the market that could handle the ingredients of any drink, as well as the ingredients to make salsa and other chopped foods. The manager intended to wait in line to buy the blender at a special sale, and the bartender asked the manager to pick an additional blender up for the bartender as well. The manager purchased three blenders, and kept one for himself. He sold one to the bartender for the sale purchase price, and gave the third to his sister as a gift. Two weeks later, the manager came across information on the Internet that the blender manufacturer was recalling the blenders encase the motors could overhead and throw sparks, causing a fire. The manager called his sister right away and left a message, but she didn't get to it in time. Her blender sparked, causing a fire in her kitchen that burned her right hand and caused major property damager. The bartender was also burned by his blender, although he didn't supper nearly as much property damage. The manager apologized to the bartender for not having hotel him about the recall. if the sister and bartender file a strict products liability action against the manager for damages, what is the likely outcome? A. The bartender and the sister will prevail against the manager, because the manager is strictly liable. B. The bartender, but not the sister, will prevail against the manager, because the manager failed to warn the bartender about the recall. C. Neither the bartender nor the sister will prevail against the manager, because the manager isn to a commercial seller. D. Neither the bartender nor the sister will prevail against the manager, because the manager did not tamper with the blender.

C. Neither the bartender nor the sister will prevail against the manager, because the manager is not a commercial seller. Discussion of correct answer:An occasional seller or, as in this case, a private party who purchases an item for somebody else is not strictly liable for defects in the product that injure the user. In this case, the manager was not a commercial distributor of the blender in question. Therefore, the manager will not be strictly liable to either the bartender, who paid him for the blender, or to his sister, to whom the manager gave the blender as a gift.

A man from State A was in a car accident with a woman from State C while both were visiting State B. The man filed a claim for $100,000 against the woman in a State B court. The woman properly removed the case to federal court. In State B court, jury verdicts were required to be in the form of special verdicts by which the jury answered specific questions of fact. However, the federal judge who heard the case followed the Federal Rules and submitted the case to the jury through a general verdict with written questions. The man objected and argued that the federal court was required to follow the state court procedure of submitting the case to the jury by special verdict. Was the federal court sitting in diversity required to submit the case to the jury by special verdict? A. Yes, because federal courts must follow state law in diversity cases. B. Yes, because the case was originally filed in state court. C. No, because the Federal Rules authorize the procedure that the federal judge used. D. No, because the method of submitting a case to aj jury is not outcome-determinative.

C. No, because the Federal Rules authorize the procedure that the federal judge used. Discussion of correct answer:The Federal Rules state that a court can submit the case to the jury by general verdict with written questions. When a Federal Rule of Civil Procedure addresses a procedural issue, that procedure applies even if the state court in the same locale would do something different. Under the Erie doctrine, federal courts follow validly enacted federal procedural rules.

A boat captain kept a personal residence on a canal near the entrance to a major waterway. The captain was often away from his home but when he was able to spend time at his residence, he liked to fish from his dock. One day, the captain returned home early from a job and found a neighbor he disliked fishing off the captain's dock. Without asking the neighbor to leave, the captain charged out of his back door and began throwing eggs at the neighbor to scare him off the dock. The neighbor called the captain "crazy" and left in a hurry, having ducked all the eggs. If the neighbor brings a tort claim against the captain, does the captain have a valid defense? A. Yes, because the captain is privileged to use force to prevent trespass to his property. B. Yes, because the captain did not intend to hit the neighbor. C. No, because the captain did not ask the neighbor to leave the dock. D. No, because placing the neighbor in apprehension of an imminent harmful touching is an assault.

C. No, because the captain did not ask the neighbor to leave the dock. Discussion of correct answer:Unless it would be futile or dangerous to make a demand, a landowner must ask a trespasser to leave prior to resorting to the use of force. Specifically, landowner defendants may use reasonable force to prevent a plaintiff from committing a tort on the defendant's property, but only after asking the plaintiff to desist, or in this case, leave. In this case, the captain did not ask the neighbor to leave, but instead resorted immediately to a physical threat. Given that he did not ask the trespassing neighbor to leave his property, the captain does not have a valid defense to a tort claim by the neighbor. Note that the use of deadly force is never permitted to protect one's property; only reasonable force may be used.

Three US citizens and siblings, two from State A and one from State B, entered into a contract with a resident of State B. However, over the course of negotiation, the parties experienced a miscommunication regarding the terms of the contract. As a result, each of the parties believed the others have breached the contract. The three siblings brought suit for breach of contract against the resident of State B. The plaintiffs' claim against the defendant was for $100,000 each. Does the federal court have jurisdiction over the action based on diversity of the litigants? A. Yes, because at least one plaintiff is diverse from the defendant. B. Yes, because diversity exists between the litigants and the amount in controversy exceeds $75,000. C. No, because there is not complete diversity among all the parties, even though tea mount in connivery exceeds $75,000. D. No, because this action does not involve a federal question, which is generally the only method for establishing original subject-matter jurisdiction in the federal trial courts

C. No, because there is not complete diversity among all the parties, even though the amount in controversy exceeds $75,000. Discussion of correct answer:Federal district courts have original jurisdiction over matters involving diverse litigants, where the amount in controversy is more than $75,000. In order for diversity to exist, there must be complete diversity among citizens of the suit; a shared citizenship between any plaintiff and any defendant will destroy diversity jurisdiction. In this example, the court will not have jurisdiction based on diversity because one of the plaintiffs and the defendant are from State B; as such, there is not complete diversity. Therefore, this is the best answer choice.

A woman sued a company for negligence because one of its drivers was involved in a car accident with her. The company timely answered the woman's complaint without speaking to the driver involved. When the company finally interviewed the driver 23 days later, the driver stated the woman's injuries were because she was driving while intoxicated at the time of the accident. Can the company now assert contributory negligence as a defense to the suit? A. Yes, because the company discovered new information. B. Yes, because the company is asserting a valid defense. C. No, because this is an affirmative defense. D. No, because 21 days have passed since service of the complaint.

C. No, because this is an affirmative defense. Discussion of correct answer:In order to avail himself or herself of certain defenses, a party must plead all affirmative defenses in his or her answer or reply to a counterclaim. An affirmative defense is one that relies on factual issues not presented in the complaint, including: (1) accord and satisfaction; (2) arbitration and award; (3) assumption of risk; (4) contributory negligence; (5) discharge in bankruptcy; (6) duress; (7) estoppel; (8) failure of consideration; (9) fraud; (10) illegality; (11) injury by fellow servant; (12) laches; (13) licenses; (14) payment; (15) release; (16) res judicata; (17) Statute of Frauds; (18) statute of limitations; and (19) waiver. Affirmative defenses are waived if not pleaded in the initial responsive pleading. Here, the company should have pled this contributory negligence defense in its initial answer.

A famous TV star hosted a popular afternoon television talk show. Her guest, a famous comedian, pointed to the host as an example of liposuction at its finest. The hose, who had actually lost weight by a grueling regime of self-deprivation and aerobics, did not appreciate the comedian's remark and therefore sued the comedian for defamation. At trial, the host calls a homemaker who saw the show in question to describe what she had heard on the broadcast. The comedian's counsel objects on the basis that the testimony would be hearsay. How should the trial judge rule on the objection? A. Sustain the objection, because the homemaker's testimony is offered to prove the truth of the matter asserted. B. Sustain the objection, because the homemaker does not have firsthand knowledge. C. Overrule the objection, because the comedian's statement is not being offered for its truth. D. Overrule the objection, because specific acts are admissible to prove character in a defamation action.

C. Overrule the objection, because the comedian's statement is not being offered for its truth. Discussion of correct answer:This evidence is non-hearsay. The hearsay rule forbids evidence of out-of-court assertions offered to prove the facts asserted in them. Proof of utterances and statements may be made with an almost infinite variety of other purposes that do not rest for their value upon the veracity of the out-of-court declarant, and hence they fall outside the hearsay classification. A few of the more common types of non-hearsay include: (1) verbal acts; (2) statements offered to show effect on the hearer or reader; and (3) declarations offered to show circumstantially the feelings or state of mind of the declarant. This evidence is not hearsay because it is not being offered for its truth, but rather, it is being used to prove that the statement was made, regardless of its truth

While driving on an intestate freeway, a suspected drug trafficker is pulled over by DEA agents who had received a tip that she was transporting 50 kg of cocaine for a drug cartel. The DEA agents search the suspected trafficker's car and find 30 kg of cocaine. They question her for over 12 hours, but she denies knowing anything and pleads with the agents to let her go. Finally, the agents offer to cut her a deal for the sake of her 2 kids. they promise to reduce the charges against her and grant her probation if she tells them the name and address of the person to whom she delivered the first 20 kg of cocaine, so they can complete the delivery with an undercover agent. The suspected trafficker provides the agents the requested information and agrees to testify at the dealer's trial. Several weeks later, when she testifies, the defendant's counsel asks her about the 12 hours during which she denied all knowledge of the cocaine cartel and suggest to the jury that she provides this information only because she had been promised leniency and in order to be able to continue to care for her children. If the state objects to the defense counsel's statement, how should the court rule on the objection? A. Sustained, because the statement is hearsay. B. Sustained, because the statement is inadmissible as part of a plea bargain negotiation. C. Overruled, because the statement goes to potential bias. D. Overruled, because the statement is relevant.

C. Overruled, because the statement goes to potential bias. Discussion of correct answer:Given that the statement is being offered not to prove the truth of the matter asserted, but to show the potential bias of the witness and the effect that the plea offer had on the suspected trafficker, it is not hearsay.

A homeowner decided to sell his house and move to another city. The homeowner found a buyer who agreed to buy the house. The two parties signed a contract, and the contract stated that the closing was to take place in one month. However, before the closing took place, the house burned down as a result of a nearby forest fire. Which party will bear the loss? A. The homeowner, because he owes the buyer an implied warranty of quality. B. The homeowner, because the fire. occurred before the closing. C. The buyer, because by the time the house burned down, the buyer had signed the contract. D. The buyer, because buyers of real property bear the risk of loss from natural disasters.

C. The buyer, because by the time the house burned down, the buyer had signed the contract. Discussion of correct answer:A purchaser of real property becomes an equitable owner at the time of the execution of a binding contract. For this reason, under the common law, the risk of loss is on the buyer upon execution of a binding contract for sale of real property. The common law rule is the majority rule. Here, the house burned down after the two parties had signed a binding contract. As such, the risk of loss will fall on the buyer, even though the closing has not yet occurred.

The President of the US entered into an executive agreement in 2035 with a neighboring country, whereby each country allowed the others ships to fish in their territorial waters. However, one of the US states had enacted a law in 1873 that allowed only ships with US registry to fish in its territorial waters, which was defined as co-extensive with the US territorial waters that extend from the state's coastline. Which of the following is the most persuasive argument? A. The state statute has precedence over the executive agreement, because the statute was first in time. B. The state statute has precedence over the executive agreement, because it had not been ratified by 2/3 of the Senate. C. The executive agreement has precedence over the state statute, because executive agreements prevail over any inconsistent state law. D. The executive agreement has precedence over the state statute, because the president has the exclusive authority to conduct foreign affairs.

C. The executive agreement has precedence over the state statute, because executive agreements prevail over any inconsistent state law. Discussion of correct answer:The president can enter into executive agreements and compacts with foreign nations. Executive agreements are the sole responsibility of the president and need not be ratified by Congress. They are valid and prevail over any inconsistent state law, although they do not prevail over federal law.

A buyer sought to purchase property for use as her home, and she obtained a loan from a lender. The buyer promised to execute a mortgage on the property in favor of the lender when she acquired the property. Three weeks later, the buyer acquired title to the property. The buyer immediately executed the promised mortgage and delivered it to the lender. However, the lender failed to record the mortgage. Unbeknownst to the lender, a creditor of the buyer had obtained a judgment against the buyer before she acquired the property. Under the law of jurisdiction, the judgment constituted a lien against any property then owned or thereafter acquired by the buyer. Who has the priority interest in the property? A. The creditor, because her lien attaches as soon as the buyer obtained title to the property. B. The creditor, if she recorded her judgment lien before the buyer executed the mortgage in favor of the lender. C. The lender, because he loaned the buyer the money to purchase the property. D. The lender, but only ifs the creditor obtained her judgment after the lender extended credit to the buyer.

C. The lender, because he loaned the buyer the money to purchase the property. Discussion of correct answer:The lender's mortgage is a purchase-money mortgage, because it secures funds advanced to purchase the property. This is so even though the loan and the mortgage were not executed concurrently. To be a purchase-money mortgage, the loan must be given as part of the same transaction in which title is acquired. However, "transaction" is broader than just the actual moment when title is transferred. The mortgage will be considered a purchase-money mortgage as long as it is part of one continuous transaction and intended to be so. Purchase-money mortgages have priority over every other type of interest that attaches to the property prior to or at the moment of acquisition through the actions of that mortgagor. Thus, the lender's purchase-money mortgage would take priority over the creditor's judgment lien, even though under the law of the jurisdiction, the creditor's lien attached at the moment the buyer acquired the property.

A restaurant owner ran a small restaurant in a suburban community, which he opened for dinner at 3:00 pm, seven days a week, even though business was often slow at that hour. One rainy afternoon, a young couple stopped in for an early dinner just as the restaurant was opening. After finishing his meal, the man left the restaurant and went to pick up the couple's car, so that his wife would not have to walk to the parking lot in the rain. Just as the man stepped off the curb onto the street, an elderly driver came around a blind corner and hit the man with his vehicle. The man was knocked to the ground and severely injured. Putting on his emergency lights, the driver got out of his car as quickly as possible and raced into the restaurant. Hearing the driver's story, the woman begged the restaurant owner to permit her to use the telephone to call an ambulance, but the restaurant owner refused, having a strict "no customer phone use" policy. The woman ran to find another open business and an ambulance arrived 15 min later. According to the man's physician, the delay in treatment allowed a blood clot to form in the man's brain, which seriously complicated his condition. If the man sues the restaurant owner, what is the restaurant owner's best defense? A. The restaurant owner's acts were a passive contributor to the man's injuries while the driver's acts were primary and active. B. The man was at fault for waling into the street. C. The man was no longer a business invitee. D. The man impliedly assumed the risk by walking into the street.

C. The man was no longer a business invitee. Discussion of correct answer:In most jurisdictions, a business owner has a duty to come to the aid of a business invitee even if he has in no way created the peril to which the invitee is exposed. Business invitees are persons who enter the defendant's business for a purpose related to the business activities or interests, and include customers and any persons accompanying them, delivery persons, salespersons (if it is reasonable for them to expect that someone on the business premises may be interested in buying from them), and job applicants (if it is reasonable for them to expect that a job may be available). Here, the man and woman were both business invitees while eating at the restaurant. If the man was still an invitee, the restaurant owner owed him a duty of reasonable care to assist him. Therefore, the restaurant owner's best defense is that the man was no longer a business invitee.

A city enacted an ordinance prohibiting any natural or legal person from emitting noise above a certain decibel level. A railroad company whose trains passed through the city on a regular basis brought a constitutional challenge to the ordinance. Which of the following is the city most likely to rely on in defending against the challenge? A. The Takings Clause. B. The affection doctrine. C. The negative implications doctrine. D. Article I, Section 10 of the US Constitution.

C. The negative implications doctrine. Discussion of correct answer:The negative implications doctrine is derived from the Commerce Clause, which gives the U.S. Congress the power to regulate interstate commerce. The negative implications doctrine states that a state may, inter alia, use its police power to pass legislation that incidentally burdens interstate commerce. The example in this question is similar to that in Huron Portland Cement Co. v. City of Detroit [362 U.S. 440 (1960)], in which the U.S. Supreme Court held that Detroit could enact a smoke abatement ordinance that affected ships traveling in interstate commerce.

A handyman owned a two-story hardware store on the Main Street of a town. The hardware store was located on a corner lot, next to a retail building owned by a lawyer. At the rear of the hardware store was a narrow alley, wide enough to accommodate a car or small truck. The handyman gave the lawyer oral permission to use the alley behind the handyman's store to access the narrow space behind the lawyer's building, where the lawyer parked his truck every weekday. The lawyer later sold his building to a dentist, who insisted on having an off street parking space. At the time of the conveyance, the lawyer negotiated with the handyman and obtained an express easement in favor of the retail building for a right-of-way through the alley. The dentist subsequently took over the retail building and parked behind the building on weekdays and Saturdays until his vehicle was vandalized, at which point he began to park it in a secured lot. After five years, the dentist sold the building to a pharmacy conglomerate that purchased the hardware store from the handyman at the same time, intending to renovate the two buildings into one large retail space. Almost immediately, the pharmacy conglomerate found a more attractive location at which to carry out its plans, and sold the hardware store and retain building to two separate small business owners. The new owner of the hardware store placed a dumpster in the alleyway, thereby blocking access to the space behind the retail building. The new owner of the retail building would like to park his car in this space. If the new owner of the retail building seeks a court order requiring the new owner of the hardware store to remove the dumpster, what is the likely outcome? A. The new owner of the retail building will prevail, because an easement by prescription exists. B. The new owner of the retail building will prevail, because the easement has not been abandoned. C. The new owner of the hardware store will prevail, because the easement merged with the fee when the pharmacy conglomerate purchased both buildings. D. the new owner of the hardware store will prevail, because the new owner of the retail building is estopped from asserting rights under the easement.

C. The new owner of the hardware store will prevail, because the easement merged with the fee when the pharmacy conglomerate purchased both buildings. Discussion of correct answer:When the dominant and servient estates come into common ownership, the easement merges with the fee and is terminated. Here, the pharmacy conglomerate purchased both the dominant estate (the retail building) and the servient estate (the hardware store). When the same owner acquired both estates, the easement was extinguished.

A buyer contracted with a seller to purchase Bill's SuperSports Bar, a popular establishment in a high-visibility commercial area. The seller sent the buyer a document via overnight mail, which stated: "I, Seller, agree to sell Bill's SuperSports Bar to Buyer for $850,000. Check for 10% down received, in escrow. Signed, Seller. Date: Nov. 15, 1999." The document was also signed by the buyer directly below the seller's signature. The buyer responded by letter in which he outlined the time and place of the closing: Jan. 3, 2000, at 10 am at the Shield Law Offices located in an office complex down the street from the SuperSports Bar. On the appointed day, both parties duly appeared to pass title to the property. The seller produced a deed conveying Bill's SuperSports Bar to the seller from another owner dated Dec. 28, 1999. The buyer learned that the other owner, in fact, owned Bill's SuperSports Bar on Nov. 15, 1999. The buyer refused to perform the contract, arguing that the seller cannot give good title to the property. In a suit by the seller for specific performance, what is the likely outcome? A. The buyer will prevail, because he did not contract with the other owner to buy the SuperSports Bar. B. The buyer will prevail, because the seller did not hold marketable title on Nov. 15, 1999. C. The seller will prevail, because he fulfilled the terms of the agreement by delivering marketable title on January 3. D. The seller will prevail, because the buyer knew that another owner owned the SuperSports Bar at the time the parties entered the contract.

C. The seller will prevail, because he fulfilled the terms of the agreement by delivering marketable title on January 3. Discussion of correct answer:Even if title is unmarketable at the time of the purchase and sale agreement, the seller does not breach the contract unless he is unable to tender marketable title at the time of the closing. Thus, the seller met the obligations of the seller on January 3 when he tendered marketable title.

An underground fad developed in which people would hold contests to determine who could toss a pig the farthest distance. The pigs, which were not fully grown, were often injured during the contests. Individuals engaging in pig tossing began to film the contests as proof of the distances the animals had travelled through the air. A documentary filmmaker obtained the trust of a band of pig-tossers and filmed their contests over period of several months. The filmmaker then edited the film, obscuring the identities of the contestants, who wished to remain anonymous. When the filmmaker distributed the film, it soon became a "cult classic", with people lining up to see it time and time again. People also began to form their own pig tossing clubs after viewing the film. When the film opened in limited release in the state, a member of PETA saw it and was outraged. Under pressure from groups such as PETA, the state legislature enacted a statute prohibiting "the depiction in film or still photography of any act of pig tossing". Violators of the statute were subject to a fine and jail time. The owner of a movie theater is arrested for running the film on 3 screens. If the owner challenges his prosecution under the state statute, how is the court likely to rule? A. The statute is constitutional, because the film is unprotected commercial speech. B. The statute is constitutional, because the film promotes violence against pigs. C. The statute is unconstitutional, because the statute chills protected speech. D. The statute is unconstitutional, because the statute is a content based regulation.

C. The statute is unconstitutional, because the statute chills protected speech. Discussion of correct answer:A government regulation that prohibits both protected and unprotected expression is considered overbroad and facially void. Here, the state statute prohibits all depictions of pig-tossing, including depictions that may be legitimate, such as pictures of the rescue of farm animals. The owner cannot be prosecuted under a statute that inhibits people from engaging in constitutionally protected expression, for fear that they might be subject to sanction. Because the statute is overbroad, chilling protected expression, it will not be upheld.

An insecticide used primarily to destroy mosquitoes and ticks is highly effective, but the compound has been found to cause nerve damage in humans, particularly the young and elderly. Satisfied that other effective insecticides existed to fight disease carriers, the US Congress enacted legislation banning use of the chemical. State Red experienced a serious public health hazard when mosquitoes in one of its counties were found to harbor a virus previously found only in Africa. The State Red Legislature immediately passed legislation permitting the use of the insecticide against any viral disease transmitted rom insects to humans. If a group representing elderly citizens of the stat challenge the State Red statute, which of the following describes the least important evidence for the court to consider when examining the constitutionality of the insecticide statute? A. Whether the regulation of insecticide use is traditionally a matter of state or local concern. B. Whether the state regulation is consistent with or supplemental to the federal regulation. C. Whether the state regulation contains a provision resolving potential conflicts with federal law. D. Whether compliance with the state and federal regulations is mutually exclusive.

C. Whether the state regulation contains a provision resolving potential conflicts with federal law. Discussion of correct answer:Whether the state insecticide regulation has such a provision is irrelevant to the court's determination as to whether the state regulation is preempted by federal law. The Supremacy Clause provides that federal law is the supreme law of the land, and inconsistent state law is preempted if expressly prohibited or if the U.S. Congress has intended the federal law to provide a comprehensive regulatory scheme. The court looks at a number of facts in determining whether state law is preempted, but the evidence provided in this answer is not a relevant factor. The Supreme Court has stated that the presence of a saving clause does not bar the ordinary working of conflict preemption principles [Geier v. Am. Honda Motor Co., 529 U.S. 861 (2000)].

A company was engaged in the business of providing crop-dusting services. The company was hired by a farmer to crop-dust at night on May 15. The crop-dusting company required all of its employees to engage in extensive training. On the night of May 15, the pilot of the crop-dusting plane noticed a large flock of birds approaching. There was no way for the pilot to avoid the birds, and he had to make an emergency landing. As the pilot maneuvered the plane for the emergency landing, the plane continued to spread the crop-dusting chemicals. Some of the chemicals ended up not eh farmer's neighbor's property and were inhaled by the neighbor, causing him to become very ill. Will the crop-dusting company be liable for the injuries caused to the neighbor? A. No, because the pilot and company exercised due care. B. No, because crop-dusting is not an abnormally dangerous activity. C. Yes, because crop-dusting is an abnormally dangerous activity. D. Yes, because the pilot was negligent in allowing the plan to continue to spread crop-dusting chemicals.

C. Yes, because crop-dusting is an abnormally dangerous activity. Discussion of correct answer:In order to successfully answer this question, you need to apply the rules regarding strict liability. One who engages in an abnormally dangerous activity is subject to strict liability regardless of exercising the utmost care to prevent the injury. In most jurisdictions, crop-dusting is indeed considered an abnormally dangerous activity. Therefore, the crop-dusting company will be liable for the neighbor's injuries.

A disgruntled employee decided to kill his boss. The employee planned to hide near his boss's house and shoot him when he arrived home from work. The employee positioned himself behind a small grove of trees about 500 yards away from the boss's front door. Shortly thereafter, the boss pulled into his driveway. As the boss exited his vehicle, the employee fired a single shot from his rifle, but the shot did not reach the boss. A neighbor heard the gunfire, however, and called the police, who later arrested the employees as he was fleeing the scene with his rifle. At the employee's trial for attempted murder, it was conceded, following the testimony of a ballistic expert, that it would have been impossible for the employee to his his boss from that distance with the rifle he had used. Is the employee guilty of attempted murder? A. No, because at that distance, he lacked the requisite intent to kill his boss. B. No, because it was legally impossible for the employee to kill the boss from that distance. C. Yes, because it was merely factually impossible for the employee to kill the boss from that distance. D. Yes, because the employee's actions were premeditated

C. Yes, because it was merely factually impossible for the employee to kill the boss from that distance. Discussion of correct answer:While legal impossibility is a defense to the inchoate crime of attempt, factual impossibility is not. Factual impossibility exists when a defendant is unable to accomplish what he intends because of some facts unknown to him. The case presented here is one of factual impossibility, as the employee had the specific intent to kill his boss and, but for a fact unknown to him (that he was positioned too far away for his rifle shot to reach the boss), he would have achieved his criminal objective. Because factual impossibility is not a defense to attempted murder, the employee can be convicted of attempted murder.

A state statute provides: "No individual or group shall stand or gather on the state capitol steps or within 50 feet of said steps for the purpose of speaking to or addressing the public at large during the weekday hours of 8:00 am to 9:00 am, or 12:00 pm to 1:30 PM." Which of the following parties is most likely to obtain a hearing in court regarding the constitutionality of the state statute? A. A national citizens' group that has lobbied successfully in the past to eliminate "restrictions on the rights of individuals in our land" B. A legislator who has introduced a bill to revoke the statute limiting speech on the grounds of the capitol. C. A state taxpayer who believes that state citizens should not have to pay the salaries of members of the governor's staff. D. A political candidate who was denied the opportunity to introduce her candidacy to the public at midday on Friday at the capitol.

D. A political candidate who was denied the opportunity to introduce her candidacy to the public at midday on Friday at the capitol. Discussion of correct answer:To establish standing, a plaintiff must show a direct injury or threat of imminent injury caused by the defendant's violation of a constitutional right. If the candidate planned to give a campaign speech on the grounds of the capitol, but was denied the opportunity, she can demonstrate direct injury. Because the state statute directly affects her free speech rights, she will likely obtain a hearing in court.

A woman filed suit against a logger in State A court claiming that he trespassed on her land and removed many of the trees without her permission. Both the woman and the logger were State A residents. Although it was unclear how many trees had been taken, the woman claimed $1 million in damages. A week later, the logger took a job in State B and moved there, establishing residency with an intent to permanently remain there. The logger then filed a motion to have the suit removed to federal court. How should the court rule? A. Grant the motion, because the man is a citizen of State B. B. Grant the motion, because the suit involves natural resources. C. Deny the motion, because the amount in controversy requirement may not be met. D. Deny the motion, because the man was a citizen of State A when the action was filed.

D. Deny the motion, because the man was a citizen of State A when the action was filed. Discussion of correct answer:The defendant has the right to remove when the plaintiff is suing the defendant in a jurisdiction that is not the defendant's home state. Removal is allowed only when at least one of the plaintiff's claims would fall within the subject-matter jurisdiction of the federal courts. Importantly, diversity has to exist at the time of the filing for the federal court to have jurisdiction through diversity. Here, at the time of filing, both parties were citizens of State A, so there was no diversity of citizenship and the claim cannot be removed.

An artist and a firefighter were neighbors who had been feeding for some time. One afternoon, the artist found some trash in her yard next to the fence that separated her and the firefighter's properties. Convinced that the firefighter had deposited the trash on her property, the artist picked I up and threw it onto the firefighter's property. The firefighter saw the artist throw the trash onto his property and immediately ran up to the artist with a sledgehammer in his hand, got to within inches of the artist's face, and shouted, "If you ever do something like that again, I'll break your leg!" The firefighter then stormed off, leaving the artist shaken by the encounter. The artist subsequently brought an action against the firefighter for intentional infliction of emotional distress. What is the likely outcome of the case? A. Judgment for the firefighter, because he did not threaten any immediate physical harm. B. Judgment for the firefighter, as long as the artist experienced no physical harm as a result of the firefighter's actions. C. Judgment for the artist, if the firefighter intended that the artist experience an apprehension of a harmful physical contact. D. Judgment for the artist, if she suffered severe emotional distress as a result of the firefighter's actions.

D. Judgment for the artist, if she suffered severe emotional distress as a result of the firefighter's actions. Discussion of correct answer: Intentional infliction of emotional distress is intentional or reckless conduct that is both extreme and outrageous and that causes the plaintiff to suffer severe emotional distress. Threatening to break the artist's leg because she threw trash onto the firefighter's property would most likely be considered extreme and outrageous conduct. While simple language that is merely offensive or insulting is generally not considered outrageous, a threat to cause someone bodily harm would most likely be considered extreme and outrageous. Such conduct is actionable if the defendant intended to cause severe emotional distress or recklessly disregarded the high probability that it would occur. In this case, the firefighter was acting intentionally, or at least recklessly. Therefore, if the artist suffered severe emotional distress, the firefighter is liable for the charged tort.

Don owned a shipping company that he used as cover to smuggle bootlegged computer game DVD-ROM discs, which he picked up at ht sport and collected in a storage container marked "Consumer electronics." Needing help, Don advertised in the classifies and eventually hired an employee, Bill, to pick up the storage container and return it to Don's main warehouse. One day, Don became convinced that the police were on to his bootleg operation. He asked Bill to help him move his surplus stock to his secondary warehouse upstate. Bill agreed and set out for the secondary warehouse. Halfway to his destination, he was pulled over by the police, who had a warrant to search inside all storage containers in the truck. The bootlegged DVD-ROM discs were discovered. Is Bill guilty of conspiracy? A. Yes, because he agreed to help the smuggler transport the container. B. Yes, because transporting the container was an overt act in furtherance of the conspiracy. C. No, because he properly withdrew from the conspiracy. D. No, because he did not agree to import bootlegged discs.

D. No, because he did not agree to import bootlegged discs. Discussion of correct answer:A conspiracy is an unlawful criminal combination between two or more people who enter into an agreement with the specific intent to commit an unlawful act or a lawful act by unlawful means. An agreement may be evidenced by conduct where the conspirators demonstrate over time that they intended to achieve the same objective and agreed to work together toward that end. Without an agreement, a conspiracy cannot exist. Here, Don hired Bill to pick up the storage container and return it to Don's main warehouse. However, there are no facts to indicate that Bill, as the employee, knew or should have known the nature of Don's smuggling business. The containers were marked "consumer electronics," and there is no indication that Bill was aware of the contents of the containers. Without more, it cannot be said that Bill agreed to help Don smuggle bootlegged DVD-ROM discs. Without this specific agreement, Bill cannot be guilty of conspiracy.

The only eyewitness to a robbery, who had been walking along the sidewalk staring at his phone and happened to look up in time to see the end of the robbery occur, made a verbal statement to police less than an hour after the event of what little of it he actually saw. The statement was transcribed and the witness reviewed the statement and signed it under oath. By the time of the trial, the witness had moved out of the country and was unavailable to testify. The prosecution moved to introduce his transcribed statement, with irrelevant or objectionable parts redacted. Should the court admit the statement into evidence? A. Yes, because the statement is not hearsay. B. Yes, because the statement is former testimony. C. No, except for purposes of rehabilitation of any witness testifying to his excited utterance/presence sense impression. D. No, because the statement is inadmissible hearsay.

D. No, because the statement is inadmissible hearsay. Discussion of correct answer:Hearsay is defined as any out-of-court statement offered into evidence to prove the truth of the matter asserted. As a general rule, hearsay is inadmissible unless it falls within a specific exception or exemption. The statement by the witness here is hearsay, and does not fall into any exception that would allow it to be admitted. The statement is not former testimony because, while it was given under oath, it was not given at a trial, hearing, or lawful deposition. Rather, the witness simply swore to the veracity of his statement to the police. The statement also would likely not qualify as an excited utterance, which is a statement relating to a startling event or condition, made while the declarant was under the stress of excitement that it caused [Fed. R. Evid. 803(2)]. An excited utterance does not need to be given contemporaneously with the event, as a present sense impression does--it can be made some time later, as long as the declarant is still under the stress of the event. However, the facts here do not support the conclusion that the witness was under the stress of excitement regarding the robbery--he was not the victim, only saw the end of the robbery, and only saw a little of what happened. Without more factual support, it does not appear that this statement would qualify as an excited utterance. Therefore, the statement would be hearsay not falling within an exception, and would be inadmissible.

A husband was arrested for physically abusing his wife, and was subsequently charged with aggravated battery. At the husband's criminal trial on the charges, the prosecutor called a doctor to the stand to testify. The doctor had provided the wife medical treatment several days after she had allegedly been battered by her husband. The doctor was to testify that, when the wife had come to his office for medical treatment, the doctor had asked her how she had sustained her injuries, and the wife replied "My husband. He has a horrible temper and completely lost control." Is the Doctor's testimony admissible? A. Yes, because the statement was made for purposes of medical diagnosis or treatment. B. Yes, because the statement was a statement of personal or family history. C. No, because the wife's statement was protected by the physician-patient privilege. D. No, because the statement was not reasonably pertinent to the diagnosis of the wife's injuries.

D. No, because the statement was not reasonably pertinent to the diagnosis of the wife's injuries. Discussion of correct answer:Under Federal Rule of Evidence 803, statements made for purposes of medical diagnosis or treatment are not excluded under the hearsay rule. Statements for purposes of medical diagnosis or treatment include statements describing medical history or past or present symptoms, pain, or sensation or the inception, as well as the general character of the cause or external source thereof, insofar as reasonably pertinent to diagnosis or treatment. However, in order for a statement to be admissible under this exception, the statement must be reasonably pertinent to the medical diagnosis and treatment of the injury. Thus, in this case, for instance, if the wife had simply stated to the doctor that she had been hit, this statement would likely have been admissible pursuant to this exception, as reasonably pertinent to the cause and treatment of the wife's injury. In contrast, under the Federal Rules, statements as to fault do not ordinarily qualify as statements reasonably pertinent to medical diagnosis or treatment. Therefore, the wife's statement to the effect that her husband "has a horrible temper and completely lost control" will not be admissible under this, or any, exception to the hearsay rule. As such, this answer is correct.

On Thanksgiving Day, the plaintiff's husband was craving a turkey for his family when he seriously cut his hand. The knife served an artery, causing the plaintiff's husband to lose a lot of blood. Plaintiff telephoned their family physician, who instructed her to drive the husband to the hospital and meet the doctor there. At the hospital, the doctor treated the plaintiff's husband. Because the plaintiff's husband had lost a considerable amount of blood, the doctor ordered an immediate blood transfusion. After determining plaintiff's husband's blood type was A positive, the doctor orally instructed the nurse, who was employed by the hospital, to obtain the necessary blood for the transfusion. The nurse followed the doctor's instructions and telephoned the hospital's blood bank department and ordered the blood. It was blood bank's normal operating procedure to consult the patient's hospital records to determine his or her blood type in response to any authorized requests. However on this occasion, someone at the blood bank, whose identity could not be determined, mislabeled the blood type for the plaintiff's husband transfusion. The blood bank by mistake sent a container with B negative blood with A positive marked on it. After being administered the wrong blood, the plaintiff's husband died. The hospital had a written rule in effect requiring that all orders for blood transfusions be made in writing by the physician in charge of the patient's treatment. Although the doctor was aware of the hospital rule, he like many other doctors frequently orders blood transfusions by oral instructions. If the plaintiff asserts a wrongful death action against the doctor, is she likely to prevail? A. Yes, because the doctor is responsible for the blood bank's error in sending the wrong blood. B. Yes, because the doctor did not follow the hospital's rule regarding blood transfusion orders. C. No, because it was the hospital's common practice not to enforce the written order rule. D. No, because the unidentified person in the blood bank who was responsible for sending the wrong blood type was not an employee of the doctor.

D. No, because the unidentified person in the blood bank who was responsible for sending the wrong blood type was not an employee of the doctor. Discussion of correct answer:As a general rule, the doctrine of respondeat superior holds that a master is vicariously liable for all tortious conduct of his servant which is within the "scope of employment." In this example, the blood bank employee (who mislabeled the blood) was not an employee or servant of the doctor. Moreover, it does not appear from the facts presented that the doctor himself was negligent in ordering the blood. Therefore, the doctor would not be responsible for the negligence of the hospital's employee. As such, this answer choice is correct.

A woman is sitting in the park on a bench reading a book. A father and his 6 year old daughter sit down on the other end of the park bench. A bee suddenly stings the little girl, who is very allergic to bees. The father panics because he can't find the EpiPen in the bag the mother packed. The woman runs to a nearby playground, asking if anyone has an EpiPen. A nanny says that she has one but needs it in case the little boy she cares for gets stung by a bee. The woman explains that a little girl was just stung and could go into shock and die without the shot soon. The nanny refused to give the woman the EpiPen, so the woman shoves the nanny to the ground, grabs the EpiPen from the nanny's diaper bag, and injects the little girl, saving her life. Should a jury convict the woman of battery? A. Yes, because the woman was not related to the little girl. B. Yes, because she intentionally applied unlawful force to the nanny's person. C. No, because the nanny, as a caregiver, owes a special duty to others and her inaction prevents her recovery. D. No, because the woman reasonably believed that showing the nanny was necessary to save the little girl's life.

D. No, because the woman reasonably believed that shoving the nanny was necessary to save the little girl's life. Discussion of correct answer:The use of reasonable force is justified when an individual reasonably believes that his criminal conduct is necessary to avoid a greater harm. In this case, the woman reasonably believed that physically pushing the nanny and taking the EpiPen was necessary to save the little girl's life, inasmuch as she was very allergic to bees. Therefore, a jury should not convict the woman of battery. Thus, this option is correct.

A corporation was organized under the laws of a foreign country and had its principal place of business in the foreign country. Because of the breakdown of centralized authority in the foreign country, the president of the corporation moved to State A and conducted all of the affairs of the corporation in State A, with the intent to return to the foreign country and conduct the affairs of the corporation there when centralized authority was restored. A businessman sued the corporation in federal court in State A for selling him defective tractors. This cause of action arose entirely in the foreign country and did not in any way relate to or arise out of the corporation's contacts with State A. State A's long arm statute extended to the limits of the Due Process Clause of the US Constitution. The corporation filed a timely motion to dismiss for lack of personal jurisdiction. The court denied the motion. Did the court err in denying the motion to dismiss? A. Yes, because the claim did not arise out of or relate to any contacts with the forum state. B. Yes, because the corporation was not incorporated in State A and maintained its principal place of business in State A only on a temporary basis. C. No, because State A's long arm statute extended to the limits of the Due Process Clause of the US Constitution. D. No, because there was a general jurisdiction over the corporation in State A.

D. No, because there was general jurisdiction over the corporation in State A. Discussion of correct answer: The facts of this question closely track the facts of Perkins v. Benguet Mining Corp. [342 U.S. 437 (1952)], in which the Supreme Court of the United States held that there was general jurisdiction in Ohio over a foreign corporation that was temporarily conducting all of its affairs in Ohio because of the Japanese occupation of the Philippines. In Daimler AG v. Bauman [134 S. Ct. 746 (2013)], the Court recently reaffirmed that Perkins was decided correctly and that it represented one of the few circumstances under which a state could exercise jurisdiction over claims that have no connection to the state.

A gossip columnist for a local newspaper was incredibly jealous of a gym teacher who she thought was better-looking and more respected in the community than the gossip columnist. When she heard that the gym teacher's most recent relationship had broken up, she published an item in her column stating that this was another in a long string of failed relationships for the gym teacher because she was clearly an immature person incapable of having healthy adult relationships, probably due to her own obsessive, narcissistic tendencies that made her extremely unpleasant to be around, and that she had at ended to stalk any romantic prospect who did not pay attention to her. When the gym teacher read the column, she was mortified. Friends began to shun her and spread word of her humiliation. The gym teacher developed such serous insomnia that she could hardly do her job and received job reprimands for the first time. She had to seek psychiatric treatment to deal with the rage and despair she felt over her misfortune. What type(s) of damages, if any may the gym teacher recover? A. Pecuniary damages. B. General damages. C. Punitive damages. D. Pecuniary, general, and punitive damages.

D. Pecuniary, general, and punitive damages. Discussion of correct answer:Pecuniary damages are monetary losses suffered due to the injury of reputation. Here, the gym teacher was forced to seek therapy, thereby incurring monetary expenses. General damages are presumed in defamation actions if there occurred non-pecuniary injury to reputation, such as humiliation and loss of friends. Finally, punitive damages may be assessed to punish and to deter future wrongful conduct. To recover punitive damages, the gym teacher must show that the gossip columnist acted with malice in publishing the item. The facts indicate that the gossip columnist did indeed act with the malice necessary to make her liable for punitive damages.

In response to the burgeoning Internet retail trade, the legislature of the State of Arden enacted a law providing that "all merchandise, other than clothing, sold through Internet transactions by entities doing business in Arden to residents of the State of Arden is subject to sales tax." The law further provided an exemption to the sales tax for any entity selling religious products online, including religion statutory, garments and print material. A businessman ran a highly profitable online retail statuary business. All of his statuary depicted secular subjects. Sales to residents of Arden accounted for 23% of the businessman's annual total revenue. Six months after the sales tax was imposed on online retail sales, the businessman filed suit in state court seeking a refund of the sales taxes collected from his Arden customers. After the state court ruled in his favor, the State of Arden appealed, and the appropriate state appellate court upheld the constitutionality of the sales tax legislation. the businessman then sought review of the case by the US Supreme court on First Amendment grounds. After theSupreme Court accepted the businessman's appeal, the legislature revised the wording of the sales tax statute to extend the tax exemption to any religious or non-religious art products, including sculpture. Assume that no other statutes apply. What is the most likely outcome of the businessman's pending appeal in the US Supreme Court? A. The appeal will be dismissed as moot. B. The appeal will be dismissed on the basis of abstention, until the slate court rules on the validity of the statute as revised. C. The appeal will be heard, because abstention is voluntary. D. The appeal will be heard on the merits.

D. The appeal will be heard on the merits. Discussion of correct answer:The sales tax legislation exempting online religious retailers raises a First Amendment issue, because the exemption may improperly promote religion in violation of the Establishment Clause. The Supreme Court has inherent judicial authority to provide uniformity in areas of constitutional law. The Supreme Court will review the federal question raised.

An elderly man who lived by himself started repainting his house. Because the elderly man was having a difficult time with the project, his son hired a contractor to do the job. Their contract provided that the contractor would receive payments after the completion of the work. When the contractor was halfway done with the job, the elderly man died. Which of the following is correct? A. The contract is discharged and the contractor will receive no payments, because he did not complete the work. B. The contract is discharged, but the contractor will receive payment for work completed. C. The contract is not discharged, but the contractor must receive half of the payment before continuing work. D. The contract is not discharged, and the contractor must complete the repainting project.

D. The contract is not discharged, and the contractor must complete the repainting project. Discussion of correct answer:The doctrine of impossibility excuses both parties from their obligations under a contract if the performance has been rendered impossible by events occurring after the contract was formed. If a contract provides that performance shall be made upon a particular person, then that person's death or incapacity will trigger the doctrine of impossibility, and the parties' obligations will be dismissed. However, in this case, the death of the elderly man does not discharge the contract between the contractor and the elderly man's son, because the elderly man is not necessary to effectuate the contract. As such, the contractor must complete the project as originally agreed.

A state legislature became concerned that the local seafood industry would be destroyed unless some new measures were undertaken to protect it. The problem stemmed from the excessive harvesting of shrimp and crab over the last several seasons in and around the ocean adjoining the state shoreline. After extensive legislative hearings, a new law was enacted that required anyone wanting to harvest shrimp or crab along the state shoreline to enter a lottery for a limited number of permits. Residents of the state who were lottery winners were given a free permit, while non-resident lottery winners must pay a $5 fee. A non-resident fisherman who possessed a valid federal fishing license authorizing him to fish in all bodies of water subject to federal regulation filed a federal action challenging the state law. What is the most likely result? A. The fisherman will not prevail, because the lottery is administered in a non-discriminatory fashion. B. The fisherman will not prevail, because the state has chosen a reasonable method of rationing a limited resource. C. The fisherman will prevail, because the state law denies him the privileges and immunities that residents of the state enjoy. D. The fisherman will prevail, because the federal license allows him to fish.

D. The fisherman will prevail, because the federal license allows him to fish. Discussion of correct answer: Pursuant to the Supremacy Clause, any state or local regulation that directly or indirectly conflicts with a valid federal law will be struck down. Here, the federal law authorizing fishing in navigable bodies of water would be valid as an exercise of the congressional power to regulate the channels, facilities, and instrumentalities of interstate commerce. The state shoreline would constitute a channel of interstate commerce. Thus, the state licensing regulation would be struck down because it directly conflicts with the federal law authorizing all federal fishing license holders to fish in the ocean.

The lack of a community center in a small town led many young people to socialize at the mall, on the streets in front of businesses, in parking lots, and on the town common. Groups of up to two dozen youths would gather in these areas, particularly at night, and residents of the town began to complain about the crows, the noise, and the garbage left behind. In response to the complaints, including a petition signed by most of the business owners on Main Street and a number of residents who claimed that they no longer felt "safe in our own town", the Board of Selectmen passed an ordinance instituting a curfew prohibiting youths 18 or younger from the downtown area after dark (9pm in the summer, 5pm in all other seasons) and further prohibiting the gathering of more than three youths in downtown area at any time. A teenager, and a resident of the town, attends a private school in neighboring city and learns of the ordinance when her brother is threatened with a citation on a public street. As part of her project for a constitutional law class, she files suit in federal court (with her father as next friend) challenging the valid of the ordinance. At the same time the teen's suit is filed, the attorney general of the state in which the town Is located brings a suit ins Tate court challenging the ordinance. If the attorney general asks the federal court to apply the abstention doctrine, which of the following would most likely result in the court's compliance with this request? A. The Board of Selectmen passes a resolution providing that the curfew ordinance will not be enforced until the state Supreme Court rules on its validity. B. The Board of Selectmen has resigned en masse over an issue unrelated to the curfew ordinance, and a new board, with views sympathetic to youth, is about to be sworn in. C. The language of the curfew ordinance is nearly identical to the language of a city ordinance struck down by the state Supreme Court on grounds that it violated the Equal Protection Clause. D. the language of the curfew ordinance is nearly identical to the language of a city ordinance struck down by the state appeals on the grounds that it was in conflict with a valid state law.

D. The language of the curfew ordinance is nearly identical to the language of a city ordinance struck down by the state appeals court on grounds that it was in conflict with a valid state law. Discussion of correct answer:The federal court may choose to abstain if it is possible that a state court decision will resolve the issues relying on unsettled state law. Under the abstention doctrine, the federal court may temporarily refuse jurisdiction, even if the case involves a federal question, if the issues can be disposed of on state law grounds. In this case, a very similar curfew ordinance in a city has been found to be in conflict with a contrary state law. Because the city ordinance has been preempted by state law, it is likely that the town ordinance will be struck down on the same grounds. The federal court can refuse to hear the case brought by the teen until the state court resolves the state law issue.

A caterer owned a large, undeveloped area of grassland and rolling hills. A mining company wished to extract coal from the land without having to purchase the property. The mining company offered to pay the caterer for the right to mine as much coal as it wished from the land in exchange for the payment of a specified sum plus a royalty per ton of coal removed. The caterer agreed and executed a document granting the requested rights for a period of 10 years, renewable upon renegotiation of the royalty payment for an additional 10 years. The mining company promptly recorded this agreement. The document further stated that the caterer agreed not to grant a similar right to any other party during the pendency of the agreement, although he could continue to use the land for his own purposes, including the removal of coal. For the next 3 years, the mining company removed coal from the land. However, a dam was subsequently constructed at the southern end of the valley in which the land was located, and the state asserted its power of eminent domain to buy all the lands in the soon-to-be flooded valley. The caterer negotiated a price and sold the land to the state. If the mining company sues the caterer to compel him to pay it a portion of the price that the state paid for the land, who will prevail? A. The caterer, because the mining company owns a license, which is not a property right protected by the Due Process Clause. B. The caterer, because the mining company owns a profit a prendre, which is not a property right protected by the Due Process Clause. C. The mining company, because it owns a license, which is a property right protected by the Due Process Clause. D. The mining company, because it owns a profit a prendre, which is a property right protected by the Due Process Clause.

D. The mining company, because it owns a profit a prendre, which is a property right protected by the Due Process Clause. Discussion of correct answer:A profit a prendre, or profit, is an interest in real property similar to an easement. The holder of a profit a prendre is not only authorized to enter the servient estate, but also has the right to remove some substance from the land. A profit a prendre is a property interest, the taking of which via eminent domain must be compensated by the state. Under the facts presented here, the mining company has a profit a prendre, which is a property interest compensable by the government when taken by eminent domain. This answer most accurately sets forth the relevant principles, and is therefore the correct answer choice.

A female horse won the most competitive horse race in the US three years ago. Since her retirement, her owner has not been able to successfully breed her. A veterinarian who specializes in new techniques for breeding animals that have not been able to conventionally conceive approached the horse's owner with an offer to purchase the horse, explaining that he had always been a devotee of horse racing and had a lifetime of ambition to own a winning horse. While the vet's explanation was truthful, he did not tell the owner that he wished to try this technique on the horse. The owner sold the horse to the vet for $50,000, a generous price for a sterile, retired racehorse, but far less than the value of the horse if she could bear offspring. Soon after the vet purchased the female horse, he extracted eggs from her ovary, fertilized them in a test tube, and implanted the fertilized egg in her womb. When the owner learns that the female horse was pregnant and that the sire was another winner of the same competitive horse race, the owner sued the vet to rescind the sale. What is the most likely outcome? A. The owner will prevail, because his ignorance of the fact that the female horse was capable of being bred successfully constitutes a unilateral mistake. B. The owner will prevail, because there was a mutual mistake. C. The vet will prevail, because attempts to breed the animal by him would be foreseeable to the owner. D. The vet will prevail, because the owner has no legal basis on which to rescind this valid contract.

D. The veterinarian will prevail, because the owner has no legal basis on which to rescind this valid contract. Discussion of correct answer:The owner and the veterinarian bargained at arm's length, and there was no confidential or fiduciary relationship between them that would have required additional disclosure. Therefore, the owner can rescind the contract only if the veterinarian made an affirmative misrepresentation. Although the veterinarian did not disclose his plans, the veterinarian made no misrepresentations. Thus, this answer is correct.

One night, a woman returned from a long business trip exhausted and ready to go to bed. However, she could not find the keys to her house in her purse. The woman saw that a window on the first floor of her neighbors' house was partially open. She did not know the neighbors well, but she was aware that they were on vacation and did not believe they would mind her crashing on their couch for the night. The woman pushed the window open, crawled through, and fell asleep on the neighbors couch. Just before the neighbors arrived home, the woman awoke and exited the house. As she was climbing back out the window, she saw. snow globe that she liked and decided to take it with her. If the woman is prosecuted for burglary, how should the court find? A. The woman is not guilty, because she did not intend to commit a felony when she entered the neighbor's house. B. The woman is not guilty, because the house was unoccupied when she entered it. C. The woman is guilty, because she broke into and entered the dwelling of another person during the night. D. The woman is guilty, because she took the snow globe with the intent to permanently deprive.

D. The woman is not guilty, because she did not intend to commit a felony when she entered the neighbor's house. Discussion of correct answer:Burglary is the breaking and entering of the dwelling of another (at common law, during the night) with the intent to commit a felony therein. In this case, the woman broke into and entered the dwelling of another (her neighbor's) at night. However, she did not do so with the intent to commit a felony, such as larceny; instead, the woman was merely looking for a place to spend the night as she was locked out of her own home. Therefore, although the woman may be guilty of trespass, she is not guilty of burglary. Note that the fact that she took the snow globe on the way out establishes only a larceny, not the intent required for burglary.

On Monday, 3 men agreed to rob a convenience store that Friday morning. The next day, one of the men, without notifying the other two, stole a struck to be used as the getaway vehicle in the robbery. The day after that, the man was arrested by the police for parole violations and was placed in jail without bail. Nevertheless, the other two men proceeded with their plan to rob the store. One of the other men, however, was a police informant who notified the police of the impending robbery. The police waited at the store and arrested the remaining man as he entered the store. Conspiracy in this jurisdiction requires proof of an overt act in addition to an agreement. Of which of the following crimes may the man who stole the truck be properly convinced? A. Theft of the truck only. B. Theft and either conspiracy to commit robbery or attempted robbery, but not both. C. Theft and attempted robbery only. D. Theft, conspiracy to commit robbery, and attempted robbery.

D. Theft, conspiracy to commit robbery, and attempted robbery. Discussion of correct answer:Under the facts presented, the man who stole the truck is clearly guilty of the theft of the truck. In addition, the defendant is guilty of conspiracy to commit robbery. Although one of his "co-conspirators" was actually a police informant who lacked the mental state required for conspiracy, the agreement between the man who stole the truck and the remaining man is sufficient to constitute an unlawful agreement, and the theft of the truck is a sufficient overt act in furtherance of the agreement. Finally, the remaining man is clearly guilty of attempted robbery, because he took a substantial step toward the commission of the robbery by entering the store. Accordingly, the man who stole the truck is also guilty of attempted robbery, because all co-conspirators are liable for all foreseeable crimes committed by any conspirator in furtherance of the original agreement.

A married couple was waiting in line for tickets to a movie. A man waiting ahead of them was quite overweight. The husband made several comments to his wife about how fat the man was, which the man overheard. He confronted the husband and demanded an apology. The husband refused. The man again demanded an apology, and threatened to punch the wife if he did not get one. To show he was serious, the man pushed the wife and waved a closed fist in her face. Fearing that his wife would suffer a heart attack from shock, the husband reached down , grabbed a pipe lying nearby on the ground, and beat the man in the head with it repeatedly. The man died from his injuries. If the husband is prosecuted for criminal homicide, should the court instruct the jury on voluntary manslaughter in addition to murder? A. No, because the husband clearly intended to kill or at least seriously injure the man. B. No, because the husband was the initial wrongdoer, having provoked the man with his rude comments. C. Yes, because the jury could reasonably conclude that the husband lacked the intent to kill or injure and was merely engaged in reckless conduct. D. Yes, because the jury could find that the husband unreasonably, but honestly, believed that deadly forced was needed to defend his wife.

D. Yes, because the jury could find that the husband unreasonably, but honestly, believed that deadly force was needed to defend his wife. Discussion of correct answer:Voluntary manslaughter is an intentional killing mitigated by adequate provocation or other circumstances negating malice aforethought. Imperfect self-defense may reduce murder to voluntary manslaughter if the defendant unreasonably, but honestly, believed that harm was imminent or that deadly force was needed. In this case, the jury could conclude that, based on the circumstances, the husband genuinely believed that the man was about to physically attack his wife, and thus that deadly force was needed, even if that belief was unreasonable. Therefore, this answer choice is correct.


संबंधित स्टडी सेट्स

Compare basic characteristics of market, traditional, command, and mixed economies.

View Set

ACC 131 - Ed Seipp Illinois State University

View Set

problem solving and data analysis

View Set